Translate

Wednesday, September 2, 2015

Bài toán: Cho $a, b, c$ là các số thực đôi một phân biệt: CMR: $$\left(\dfrac{a+b}{a-b}\right)^2+\left(\dfrac{b+c}{b-c}\right)^2+\left(\dfrac{c+a}{c-a}\right)^2+\dfrac{1}{\left(\dfrac{a+b}{a-b}\right)^2+\left(\dfrac{b+c}{b-c}\right)^2+\left(\dfrac{c+a}{c-a}\right)^2}\ge \dfrac{5}{2}$$
Lời giải: 
Đặt: $x=\dfrac{b+c}{b-c}, y=\dfrac{c+a}{c-a}, z=\dfrac{a+b}{a-b}$. Khi đó, ta có: $$(x+1)(y+1)(z+1)=(x-1)(y-1)(z-1)\Leftrightarrow xy+yz+zx=-1$$ Và ta cần CM: $$x^2+y^2+z^2+\dfrac{1}{x^2+y^2+z^2}\ge \dfrac{5}{2}\\\Leftrightarrow (x+y+z)^2+2+\dfrac{1}{(x+y+z)^2+2}\ge \dfrac{5}{2}$$ Đặt $t=x+y+z$. Khi đó xét hàm $f(t)=t^2+\dfrac{1}{t^2+2}$ là ta có ngay đpcm


Friday, August 7, 2015

Bài toán (Nguyễn Đình Thi) Chứng minh rằng với mọi $a,b,c\in\mathbb{R}$ thì $$\left(\dfrac{a^2+bc}{a^2-bc}\right)^2+\left(\dfrac{b^2+ca}{b^2-ca}\right)^2+\left(\dfrac{c^2+ab}{c^2-ab}\right)^2\ge 2$$ 
Lời giải: 
Đặt $x=\dfrac{a^2+bc}{a^2-bc}, y=\dfrac{b^2+ca}{b^2-ca},z=\dfrac{c^2+ab}{c^2-ab}$ Khi đó, ta có: $$(x+1)(y+1)(z+1)=(x-1)(y-1)(z-1)\Leftrightarrow xy+yz+zx=-1$$ Áp dụng BDT $$x^2+y^2+z^2\ge -2(xy+yz+zx)$$ Ta có $$\left(\dfrac{a^2+bc}{a^2-bc}\right)^2+\left(\dfrac{b^2+ca}{b^2-ca}\right)^2+\left(\dfrac{c^2+ab}{c^2-ab}\right)^2\ge 2$$ Ta có đpcm.


Problem (Nguyen Dinh Thi) Let $a, b, c$ be real numbers. Prove that: $$\left(\dfrac{a^2+bc}{a^2-bc}\right)^2+\left(\dfrac{b^2+ca}{b^2-ca}\right)^2+\left(\dfrac{c^2+ab}{c^2-ab}\right)^2\ge 2$$ Proof: 
Let $x=\dfrac{a^2+bc}{a^2-bc}, y=\dfrac{b^2+ca}{b^2-ca},z=\dfrac{c^2+ab}{c^2-ab}$ Then,  we have: $$(x+1)(y+1)(z+1)=(x-1)(y-1)(z-1)\Leftrightarrow xy+yz+zx=-1$$ Using this inequality: $$x^2+y^2+z^2\ge -2(xy+yz+zx)$$ We have: $$\left(\dfrac{a^2+bc}{a^2-bc}\right)^2+\left(\dfrac{b^2+ca}{b^2-ca}\right)^2+\left(\dfrac{c^2+ab}{c^2-ab}\right)^2\ge 2\;\;\text{(Q.E.D)}$$ 

Thursday, July 30, 2015

Bài toán: Cho $n$ là số nguyên dương. Tìm số đa thức $P(x)$ có hệ số thực thuộc $\{ 0, 1, 2, 3\}$ thỏa mãn $P(2)=n$.
Lời giải: 
Giả sử: $$P(x)=a_0+a_1x+a_2x^2+a_3x^3+...., \; \text{với } a_i\in\{ 0, 1, 2, 3\}$$ Do $P(2)=n$ nên $$a_0+2a_1+4a_2+...+2^ia_i+...=n\;\;\;\;\;\;\;\;\;\;\;\;(*)$$ Ta xây dựng hàm sinh cho phương trình $(*)$: $$\begin{aligned} G(x)&=\prod_{i\ge 0}\left( 1+x^{2^k}+x^{2.{2^k}}+x^{3.2^k}\right)\\&=\prod_{i\ge 0}\dfrac{1-(x^{2^k})^4}{1-x^{2^k}}\\&=\prod_{i\ge 0}\dfrac{1-x^{2^{k+2}}}{1-x^{2^k}}\\&=\dfrac{1}{(1-x)(1-x^2)}\\&=\dfrac{1}{4(1-x)}+\dfrac{1}{4(1+x)}+\dfrac{1}{2(1-x)^2}=\sum_{i\ge 0}\left( \dfrac{1}{4}+\dfrac{1}{4}(-1)^i+\dfrac{1}{2}C^{i}_{i+1}\right) x^i\end{aligned}$$ Số đa thức thỏa mãn bài toán cũng chính là hệ số của $x^n$ trong khai triển trên: $$\left[\dfrac{2n+3+(-1)^n}{4}\right]=\left[\dfrac{n}{2}\right]+1$$

Sunday, July 26, 2015

Bài toán: (Olympic 30/04/2015 - Khối 11) Cho $a,b,c$ là các số nguyên tố. Đặt $x=a+b-c,y=a+c-b,z=b+c-a$. Giả sử rằng $x^2=y$ và hiệu $\sqrt{z}-\sqrt{y}$ là bình phương của một số nguyên tố. Tính giá trị biểu thức: $$T=(a+2)(b-10)(c+2)$$
Lời Giải: 
Gọi $p$ là số nguyên tố sao cho $$\begin{aligned}&\sqrt{z}-\sqrt{y}=p^2\\\Leftrightarrow &z+y-2\sqrt{yz}=p^4\\\Leftrightarrow &2c-2\sqrt{(c+a-b)(b+c-a)}=p^4\end{aligned}$$ Từ đây suy ra $2\;|\; p\Rightarrow p=2$. Ta có hệ: $$\begin{cases}c-\sqrt{(c+a-b)(b+c-a)}=8\\(a+b-c)^2=c+a-b\end{cases}$$ Ta biến đổi PT đầu: $$\begin{aligned}& (c+a-b)(a+b-c)=(c-8)^2\\\Leftrightarrow &c^2-(a-b)^2=(c-8)^2\\\Leftrightarrow & (c-8)^2+(a-b)^2=c^2\end{aligned}$$ Xem đây là phương trình Pythargores, chú ý rằng $\gcd{(c-8, c, a-b)}=1$ (vì $a,b,c$ nguyên tố) nên:

     + Trường hợp 1: $\begin{cases} c-8=2mn\\a-b=m^2-n^2\\c=m^2+n^2\end{cases}\; (m,n\in\mathbb{N})$ Do $c$ nguyên tố mà lúc này $2\;|\; c$ nên $c=2$. Như vậy: $$(a+b-2)^2=a-b+2$$ Vì $\gcd{(c-8, a-b, c)}=1$ nên $a,b$ khác tính chẵn lẻ. Tuy nhiên khi $a=2\vee b=2$ thì hệ đều không thỏa mãn.

     + Trường hợp 2: $\begin{cases}a-b=2mn\\c-8=m^2-n^2\\c=m^2+n^2\end{cases}(m,n\in\mathbb{N})$ Từ đây suy ra luôn $n=2\Rightarrow \begin{cases}c=m^2+4\\a-b=4m\end{cases}$ Thay vào hệ trên ta được: $$(a+b-m^2-4)^2=(m+2)^2$$
          $\bullet $ Khả năng 1: $a+b-m^2-4=m+2\Leftrightarrow a+b=m^2+m+6$. Kết hợp với $a-b=4m$ suy ra: $$\begin{cases}a=\dfrac{(m^2+5m+6}{2}\\b=\dfrac{m^2-3m+6}{2}\end{cases}$$ Đặt $m=2k+1,k\in\mathbb{Z}$ Suy ra $a=(k+2)(2k+3)$ Do $a$ nguyên tố nên $$2k+1=^+_- 1\vee k+2=^+_-1\Rightarrow k=-3\Rightarrow a=3, b=23, c=29\Rightarrow \boxed{T=2015}$$
     
         $\bullet $ Khả năng 2: $a+b-m^2-4=-m-2\Leftrightarrow a+b=m^2-m+2$. Suy ra: $$\begin{cases} a=\dfrac{m^2+2m+2}{2}\\b=\dfrac{m^2-5m+2}{2}\end{cases}$$ Đặt $m=2k+1, k\in\mathbb{Z}$ Suy ra $a=(k+1)(2k+3)$ Tương tự ta cũng có $2k+3=^+_- 1\vee k+1=^+_- 1$ Tuy nhiên  lúc này ta không nhận được giá trị $a, b, c$ nguyên tố nào.

Tóm lại $$\boxed{T=2015}$$

Monday, July 20, 2015

Bài toán: Tìm tất cả số nguyên dương $n$ sao cho $2^n-1$ chia hết cho $3$ và $\dfrac{2^n-1}{3}$ là ước của một số nguyên có dạng $4m^2+1$.

Lời giải:

Bổ đề 1: Với mỗi số nguyên dạng $4k+3$ thì luôn tồn tại một ước nguyên tố $p\equiv 3\pmod{4}$.
Chứng minh: 
Giả sử $n$ là số nguyên dạng $4k+3$. G/s luôn $n=p_1^{a_1}p_2^{a_2}...p_j^{a_j}$, với $p_i$ là các số nguyên tố, $a_i\in\mathbb{N^*}, i=\overline{1, k}$. Vì $n=4k+3$ nên $n$ là số lẻ. Nếu $p_i\equiv 1\pmod{4}$ thì $n\equiv 1\pmod{4}$, vô lý. Vậy phải tồn tại $p_l$ sao cho $p_l\equiv 3\pmod{4}$. Bổ đề 1 được chứng minh.

Bổ đề 2: Nếu $p$ là số nguyên tố dạng $4k+3$ và tồn tại $x, y\in\mathbb{Z}$ thỏa $p\;|\; x^2+y^2$ thì $\begin{cases}p\;|\; x\\p\;|\; y\end{cases}$
Chứng minh: 
 Ta xét trong TH $\begin{cases}p\;\not |\; x\\p\;\not |\; y\end{cases}$. Sử dụng định lý Fermat, ta có: $$\begin{cases} x^{p-1}\equiv 1\pmod{p}\\y^{p-1}\equiv 1\pmod{p}\end{cases}\Rightarrow \begin{cases}x^{4k+2}\equiv 1\pmod{p}\\y^{4k+2}\equiv 1\pmod{p}\end{cases}\Rightarrow x^{4k+2}+y^{4k+2}\equiv 2\pmod{p}$$ Điều này mâu thuẫ với giả thiết $p\;|\; x^2+y^2$. Vậy bổ đề 2 được chứng minh.

Quay lại bài toán:
Ta có: $2\equiv -1\pmod{3} $ nên $2^n\equiv 1\pmod{3}\Leftrightarrow n $ chẵn. Giả sử: $n=2^k.h,\; h$ lẻ. ta sẽ CM $h=1$. Thật vậy, g/s $h>1$. Ta có: $$2^h-1\;|\; 2^n-1\;|\; 3.4m^2+3$$ Do xét $h $ lẻ nên ta xét với $2^h-1\;|\; 4m^2+1$. Do $2^h-1\equiv 3\pmod{4}$ nên theo bổ đề 1, tồn tại một ước nguyên tố của $2^h -1$ dạng $2^l-1$ Sử dụng bổ đề 2, ta có: $$\begin{cases}2^l-1\; |\; 4m^2\\2^l-1\;|\;1\end{cases}\Leftrightarrow l=1, \text{vô lý}$$ Do vậy $n=2^k, k\in\mathbb{N}$. Ta sẽ CM đây cũng chính là KQ cần tìm. Ta phân tích: $$2^{2^k}-1=(2^{2^0}+1)(2^{2^1}+1)(2^{2^2}+1)...(2^{2^{k-1}}+1)$$ Chú ý rằng $3\;|\; 2^n-1$ nên ta xét với $\prod_{i=1}^{k-1}(2^{2^i}+1)$ Xét hệ thặng dư: $$\begin{cases} x\equiv 2^{2^0-1}\pmod{2^{2^1}+1}\\x\equiv 2^{2^1-1}\pmod{2^{2^{2}}+1}\\...\\x\equiv 2^{2^{i-1}-1}\pmod{2^{2^i}+1}\\...\\x\equiv 2^{2^{k-2}-1}\pmod{2^{2^{k-1}}+1}\end{cases}\Leftrightarrow \begin{cases}4x^2\equiv -1\pmod{2^{2^1}+1}\\4x^2\equiv -1 \pmod{2^{2^2}+1}\\...\\4x^2\equiv -1\pmod{2^{2^i}+1}\\...\\4x^2\equiv -1\pmod{2^{2^{k-1}}+1}\end{cases}$$. Chú ý rằng $\gcd(2^{2^i}+1; 2^{2^j}+1)=1, i\neq j$ nên theo định lý thặng dư Trung Hoa, hệ này có nghiệm duy nhất: $$4x^2\equiv -1\pmod{\prod_{i=1}^{k-1}(2^{2^i}+1)}\Leftrightarrow 4m^2+1\;\vdots \;\dfrac{2^n-1}{3}$$
Vậy: $n=2^k, k\in\mathbb{N}$

Saturday, July 18, 2015

HÀM LỒI, LÕM. HÀM BÁN LỒI, BÁN LÕM VÀ NGUYÊN LÝ BIÊN

HÀM LỒI, LÕM. HÀM BÁN LỒI, BÁN LÕM
VÀ NGUYÊN LÝ BIÊN

I. Định nghĩa:

    1. Hàm lồi, lõm:
       Hàm số $f$ được gọi là lồi trên đoạn $[\alpha; \beta ]\subset\mathbb{R}$ nếu với mọi $x, y\in [\alpha; \beta ]$ và với mọi $a,b\ge 0$ thỏa $a+b=1$ thì: $$f(ax+by)\ge af(x)+bf(y)$$ 
       Ngược lại, $f$ được gọi là lõm trên đoạn $[\alpha; \beta ]\subset\mathbb{R}$ nếu với mọi $x, y\in [\alpha; \beta ]$ và với mọi $a,b\ge 0$ thỏa $a+b=1$ thì: $$f(ax+by)\le af(x)+bf(y)$$ 

 Chú ý: Ta thừa nhận kết quả sau:

     Nếu hàm $f$ khả vi hai lần trên $[\alpha ;\beta ]$ thì $f$ lồi trên $[\alpha ;\beta ]$ khi $f''\le 0,\forall x\in [\alpha; \beta ]$. Ngược lại, $f$ lõm trên $[\alpha; \beta ] $ khi $f''\ge 0,\forall x\in [\alpha ;\beta ]$

    2. Hàm bán lồi, bán lõm:

       Hàm số $f$ được gọi là bán lồi, bán lõm trên $[\alpha ; \beta ]\subset \mathbb{R}$ nếu tồn tại hằng số $\gamma\in [\alpha ; \beta ]$ duy nhất sao cho $f$ lồi trên $[\alpha ; \gamma ]$ và lõm trên $[\gamma ; \beta ]$ hoặc ngược lại.

II. Tính chất:

                                                                         
    1. Nếu $f$ là hàm lồi trên $[\alpha ; \beta ]$ thì ta có: $\min f=\min\{ f(\alpha ); f(\beta )\}$
    2. Nếu $f$ lõm trên $[\alpha ; \beta ]$ thì ta có: $\max f=\max\{ f(\alpha ); f(\beta )\}$
    3. Nếu $f$ là một hàm lõm trên đoạn $[\alpha ; \beta ]$ thì khi đó với mọi $x,y, z\in\mathbb{R}$ thỏa mãn đồng thời $\beta \ge x\ge z\ge y\ge \alpha$ và $x+y-z\ge\alpha  $ thì $$f(x)+f(y)-f(z)\ge f(x+y-z)$$ 
 Chứng minh: 
 Đặt $t=x-z$, dễ thấy $0\le t\le x-y\Rightarrow t=k(x-y), k\in [0; 1]$. Ta có: $$\begin{aligned}f(x)+f(y)-f(z)-f(x+y-z)&=f(x)+f(y)-f(x-t)-f(y+t)\\&=f(x)+f(y)-f\left(x-k(x-y)\right)-f\left( y+k(x-y)\right)\\&=f(x)+f(y)-f\left((1-k)x+ky\right)-f\left( kx+(1-k)y\right) \\&\ge f(x)+f(y)-(1-k)f(x)-kf(y)-kf(x)-(1-k)f(y)=0\;\;\;\text{(Q.E.D)}\end{aligned}$$ 

   4. Nếu $f$ lồi trên  trên đoạn $[\alpha ; \beta ]$ thì khi đó với mọi $x,y, z\in\mathbb{R}$ thỏa mãn đồng thời $\beta \ge x\ge z\ge y\ge \alpha$ và $x+y-z\ge\alpha$ thì $$f(x)+f(y)-f(z)\le f(x+y-z)$$ 

   5. Nếu $f$ lõm trên $[\alpha ; \beta ]$ thì với dãy số $x_n \in [\alpha ; \beta ]$ thỏa $\sum_{i=1}^nx_i\le \beta+(n-1)\alpha$ thì khi đó: $$f(x_1)+f(x_2)+...+f(x_n)-(n-1)f(a)\le f(x_1+x_2+...+x_n-(n-1)\alpha )$$
 Chứng minh: 
 Ta chứng minh bằng qui nạp theo $n$ 
 Với $n=1$, hiển nhiên đúng.
 Giả sử tính chất 5 đúng tới $n$. Ta xét khi $n+1$.
 Không giảm tính tổng quát, giả sử $x_{n+1}=\max\{x_1; x_2;...; x_{n+1}\}$. Sử dụng giả thiết qui nạp ta chỉ cần chứng minh: $$f(x_1+x_2+...+x_n-(n-1)\alpha )+f(x_{n+1})\le f(\alpha )+f(x_1+x_2+...+x_{n+1}-n\alpha )$$ Do $x_{n+1}=\max\{x_1, x_2,..., x_{n+1}\}$ và $x_i\in [\alpha; \beta ]$ nên: $$\begin{aligned}f(\alpha )+f(x_1+x_2+...+x_{n+1}-n\alpha )&\ge f(x_1+x_2+...+x_{n+1}-n\alpha +\alpha -x_{n+1})+f(x_{n+1})\\&=f(x_1+x_2+...+x_n-(n-1)\alpha )+f(x_{n+1})\end{aligned}$$ Như vậy, khẳng định đúng với $n+1$. Ta có đpcm.

  6. Nếu $f$ lồi trên $[\alpha ; \beta ]$ thì với dãy số $x_n \in [\alpha ; \beta ]$ thỏa $\sum_{i=1}^nx_i\le \beta+(n-1)\alpha$ thì khi đó: $$f(x_1)+f(x_2)+...+f(x_n)-(n-1)f(a)\ge f(x_1+x_2+...+x_n-(n-1)\alpha )$$

 7. (BDT tiếp tuyến)
      $*$ Nếu $f$ lõm trên $[\alpha ; \beta ]$ thì với mọi $x, x_0\in [\alpha ; \beta ]$ ta có : $$f(x)\ge f'(x_0)(x-x_0)+f(x_0)$$
      $*$ Nếu $f$ lồi trên $[\alpha ; \beta ]$ thì với mọi $x, x_0\in [\alpha ; \beta ]$ ta có : $$f(x)\le f'(x_0)(x-x_0)+f(x_0)$$

  $*$ Đặc biệt, khi $f(x)=ax+b, a\neq 0$ thì đồ thị $f$ là một đoạn thẳng nên: $$\min\{f(\alpha ); f(\beta )\}\le f(x)\le \max\{f(\alpha ); f(\beta )\}$$

Định lý 1: Cho $(x_n)$ là dãy các số thực thỏa:
               i) $(x_n)$ là dãy không giảm.
               ii) $x_n\in [\alpha ; \beta ]$
               iii) $x_1+x_2+...+x_n=k=\text{const}$
               iv) $f$ là hàm trên $[\alpha ; \beta ]$ thỏa $f$ lồi trên $[\alpha ; \gamma ]$ và lõm trên$[\gamma ; \beta ]$.
Đặt $F=f(x_1)+f(x_2)+...+f(x_n)$. Khi đó: 
       + $\min F\Leftrightarrow x_1=x_2=...=x_{k-1}=a; x_{k+1}=...=x_n\in [\alpha ; \beta ]$
       + $\max F\Leftrightarrow x_1=x_2=...=x_{k-1}\in [\alpha ; \beta ]; x_{k+1}=...=x_n=b$

Định lý 2: Cho $(x_n)$ là dãy các số thực thỏa:
               i) $(x_n)$ là dãy không giảm.
               ii) $x_n\in [\alpha ; \beta ]$
               iii) $x_1+x_2+...+x_n=k=\text{const}$
               iv) $f$ là hàm trên $[\alpha ; \beta ]$ thỏa $f$ lõm trên $[\alpha ; \gamma ]$ và lồi trên$[\gamma ; \beta ]$.
Đặt $F=f(x_1)+f(x_2)+...+f(x_n)$. Khi đó: 
       + $\max F\Leftrightarrow x_1=x_2=...=x_{k-1}=a; x_{k+1}=...=x_n\in [\alpha ; \beta ]$
       + $\min F\Leftrightarrow x_1=x_2=...=x_{k-1}\in [\alpha ; \beta ]; x_{k+1}=...=x_n=b$

 Định lý 3: Cho $(x_n)$ là dãy các số thực thỏa:
               i) $(x_n)$ là dãy không giảm.
               ii) $x_n\in\mathbb{R}$
               iii) $x_1+x_2+...+x_n=k=\text{const}$
               iv) $f$ là hàm trên $[\alpha ; \beta ]$ thỏa $f$ lồi trên $(-\infty; \gamma ]$ và lõm trên$[\gamma ; +\infty )$.
Đặt $F=f(x_1)+f(x_2)+...+f(x_n)$. Khi đó: 
       + $\min F\Leftrightarrow x_1\le x_2=x_3=...x_n$
       + $\max F\Leftrightarrow x_1=x_2=...=x_{n-1}\le x_n$

Định lý 4: Cho $(x_n)$ là dãy các số thực thỏa:
               i) $(x_n)$ là dãy không giảm.
               ii) $x_n\in\mathbb{R}$
               iii) $x_1+x_2+...+x_n=k=\text{const}$
               iv) $f$ là hàm trên $[\alpha ; \beta ]$ thỏa $f$ lõm trên $(-\infty; \gamma ]$ và lồi trên$[\gamma ; +\infty )$.
Đặt $F=f(x_1)+f(x_2)+...+f(x_n)$. Khi đó: 
       + $\max F\Leftrightarrow x_1\le x_2=x_3=...x_n$
       + $\min F\Leftrightarrow x_1=x_2=...=x_{n-1}\le x_n$

II. Các ví dụ: 

 Ví dụ 1: (IMO Shorlist 1993, Viet Nam) Cho $a, b, c, d$ là các số thực không âm thỏa $a+b+c+d=1$. CMR: $$abc+abd+acd+bcd\le \dfrac{1}{27}+\dfrac{176}{27}abcd$$
Lời giải:
  Giả sử $a=\min\{a,b,c,d\}\Rightarrow a\le\dfrac{1}{4}$. BDT cần chứng minh viết lại dưới dạng: $$\left(bc+cd+bd-\dfrac{176}{27}bcd\right)a+bcd\le\dfrac{1}{27}\;\;\;\;\;\;\;\;\;\;(1)$$ Xem VT của $(1)$ như một hàm số theo $a$. Ta xét các trường hợp: 
      Trường hợp 1: $bc+cd+bd-\dfrac{176}{27}bcd=0.$ Khi đó:  $$VT_{(1)}=bcd\le\dfrac{(b+c+d)^3}{27}\le\dfrac{(a+b+c+d)^3}{27}=\dfrac{1}{27}$$
      Trường hợp 2: $bc+cd+bd-\dfrac{176}{27}bcd<0.$ Khi đó:  $$VT_{(1)}\le bcd\le \dfrac{1}{27}$$
      Trường hợp 3: $bc+cd+bd-\dfrac{176}{27}bcd>0.$ Khi đó: $$VT_{(1)}\le \dfrac{bc+cd+bd}{4}-\dfrac{17}{27}bcd$$ Sử dụng BDT Schur, ta có: $$\dfrac{bc+cd+bd}{4}-\dfrac{17}{27}bcd\le \dfrac{13}{324}(bc+cd+bd)+\dfrac{17}{576}\le \dfrac{13(b+c+d)^2}{324.3}+\dfrac{17}{576}=\dfrac{1}{27}$$
 Phép chứng minh hoàn tất.
    Lưu ý: Tùy theo bài toán ra cần lựa đại lượng thay đổi phù hợp sao cho thuận lợi trong việc công phá bài toán. Đối với bài toán này, ta có thể cố định $c, d$ thì do $a+b+c+d=1$ nên $a+b$ cũng cố định, lúc này khảo sát $VT_{(1)}$ theo $ab$ ta cũng thu dược đpcm.

Ví dụ 2: (Vasile Cirtoaje) Cho $a, b, c, d$ là các số thực không âm $a, b, c, d$ thỏa $a^2+b^2+c^2+d^2=1$. CMR: $$(1-a)(1-b)(1-c)(1-d)\ge abcd$$
Lời giải:
 Giả sử $a=\min\{a,b,c,d\}\Rightarrow a\le\dfrac{1}{2}$. Ta viết BDT cần chứng minh dưới dạng: $$[(1-b)(1-c)(1-d)+bcd]a-(1-b)(1-c)(1-d)\le 0\;\;\;\;\;\;\;\;\;\;\;\;\;(1)$$ Xem VT của $(1)$ như một hàm theo $a$. Ta có hai trường hợp:
     Trường hợp 1: $(1-b)(1-c)(1-d)+bcd=0$, hiển nhiên đúng.
     Trường hợp 2: $(1-b)(1-c)(1-d)+bcd>0$. Khi đó: $$[(1-b)(1-c)(1-d)+bcd]a-(1-b)(1-c)(1-d)\le \dfrac{bcd-(1-b)(1-c)(1-d)}{2}$$ Đặt $\begin{cases}p=b+c+d\\q=bc+cd+db\\r=bcd\end{cases}$. Khi đó, ta cần CM:$$\dfrac{2r+p-q-1}{2}\le 0$$ Chú ý là lúc này $p^2-2q=\dfrac{3}{4}$ nên ta cần CM: $$4r-\dfrac{1}{4}\le (p-1)^2$$ Theo AM-GM: $p\ge 3\sqrt[3]{r}$ Từ đây chỉ việc khảo sát hàm $g ( r ) =4r-9\sqrt[3]{r^2}+6\sqrt{r}-\dfrac{5}{4}, r\le \dfrac{1}{8}$ là có đpcm. 


 Ví dụ 3: (Crux Mathemmaticorum) Cho $a,b,c$ là các số không âm. CMR: $$\sqrt{1+\dfrac{48a}{b+c}}+\sqrt{1+\dfrac{48b}{c+a}}+\sqrt{1+\dfrac{48c}{a+b}}\ge 15$$ Lời giải:

Đặt $x=\dfrac{2a}{b+c}; y=\dfrac{2b}{c+a}; z=\dfrac{2c}{a+b}$. Khi đó, ta có: $$xy+yz+zx+xyz=4$$ Và ta cần CM: $$\sqrt{1+24x}+\sqrt{1+24y}+\sqrt{1+24z}\ge 15\;\;\;\;\;\;\;\;\;\;\;\;\;\;\;(1)$$ Không giảm tính tổng quát, giả sử: $x=\max\{ x,y, z\}\Rightarrow x\ge 1$. Xem VT của $(1)$ như một hàm $f$ theo $x$. Dễ thấy $f$ đồng biến nên $f$ đạt giá trị nhỏ nhất khi $x=1$. Lúc này, ta có $y+z+2yz=4\Leftrightarrow y=\dfrac{4-z}{1+2z}$ và ta cần tìm GTNN của $$\sqrt{1+24y}+\sqrt{1+24z}=\sqrt{1+24.\dfrac{4-z}{1+2z}}+\sqrt{1+24z}$$ Thực hiện khảo sát hàm $g(z)=\sqrt{1+24.\dfrac{4-z}{1+2z}}+\sqrt{1+24z}$ với $z\in [0; 1]$ ta dễ dàng thấy $g(z)\ge 10$ Từ đó suy ra: $$\sqrt{1+24x}+\sqrt{1+24y}+\sqrt{1+24z}\ge 15\;\text{  (Q.E.D)}$$

Ví dụ 4: Cho các số $a,b,c$ không âm. CMR: $$\left( \dfrac{4a}{b+c}+1\right)\left( \dfrac{4b}{c+a}+1\right)\left(\dfrac{4c}{a+b}+1\right)\ge 25$$ Lời giải:

Đặt $x=\dfrac{2a}{b+c}, y=\dfrac{2b}{c+a}, z=\dfrac{2c}{a+b}$. Khi đó: $$xy+yz+zx+xyz=4$$
Và ta cần CM: $$(1+2x)(1+2y)(1+2z)\ge 25$$Nếu ta xem VT của BDT này như một hàm theo $z$ thì rõ ràng đây là một hàm bậc nhất và đồng biến. Theo đó. GTNN của hàm này đạt được khi $z=0$. Lúc này $xy=4$ và ta cần chỉ ra: $$(1+2x)(1+2y)\ge 25\Leftrightarrow x+y\ge 8$$ Điều này là hiển nhiên theo AM-GM. Bài toán được chứng minh.

Ví dụ 5: (USA MO 2000) Cho $a,b, c$ là các số thực không âm. CMR: $$\dfrac{a+b+c}{3}-\sqrt[3]{abc}\le\max\{(\sqrt{a}-\sqrt{b})^2; (\sqrt{b}-\sqrt{c})^2; (\sqrt{c}-\sqrt{a})^2\}$$ Lời giải:

Không mất tính tổng quát, giả sử $a\le b\le c$. Khi đó, ta cần CM: $$a+b+c-3\sqrt[3]{abc}-3(\sqrt{c}-\sqrt{a})^2\ge 0\;\;\;\;\;\;\;\;\;\;\;\;\;\;\;\;\;(1)$$ Xem VT của $(1)$ là một hàm theo  $b$. Ta tính được $$f''(b)=\dfrac{\sqrt[3]{ca}}{3\sqrt[3]{b^5}}>0$$
 Do đó $f$ là hàm lõm. Như vậy GTLN của $f$ đạt được khi $b=a$ hoặc $b=c$. Ta xét khi $a=b$. Khi đó ta cần CM: $$a+2c+3\sqrt[3]{ca^2}\ge 6\sqrt{ca}$$ Điều này là đúng theo AM-GM.

Nếu mở rộng BDT trên ta tìm được chặn dưới của đại lượng TBC-TBN là $\dfrac{1}{2}\min_{1\le i<j\le n}\{(\sqrt{a_i}-\sqrt{a_j})^2\}$. Đây chính là bài toán BDT trong kì thi MOSP 2000: 

 Cho $a_1, a_2,...,a_n$ là các số không âm. CMR: $$\dfrac{1}{n}\sum_{i=1}^n a_i-\sqrt[n]{\prod_{i=1}^n a_i}\ge \dfrac{1}{2}\min_{1\le i<j\le n}\{(\sqrt{a_i}-\sqrt{a_j})^2\}$$ 

Ví dụ 6: Cho $a, b, c\ge 0$ có tổng bằng $1$. CMR: $$\sqrt{\dfrac{1-a}{1+a}}+\sqrt{\dfrac{1-b}{1+b}}+\sqrt{\dfrac{1-c}{1+c}}\le 1+\dfrac{2}{\sqrt{3}}$$ Lời giải: 
Giả sử $a\le b\le c$. Xét hàm số: $$ f(x)=\sqrt{\dfrac{1-x}{1+x}}, x\in [0; 1]$$ Ta tính được: $$f''(x)=\dfrac{1-2x}{(x+1)^4\sqrt{\left(\dfrac{1-x}{1+x}\right)^3}}$$ Ta thấy $f''$ đổi dấu khi qua $x=1/2$ nên $f$ lõm trên $\left[0; \dfrac{1}{2}\right]$ và lồi trên $\left[\dfrac{1}{2}; 1\right]$. Do đó theo định lý 2, VT đạt cực đại khi: $$\begin{cases}a=0, b=c\\a+b+c=1\end{cases}\vee \begin{cases}a=b=0\\c=1\end{cases}$$ Giải các hệ này ta dễ dàng suy ra đpcm.

Tổng quát hóa bài toán ví dụ 6, ta thu được bài toán của tạp chí Crux Mathemmaticorum: 
Cho $x_1, x_2,..., x_n\;\; (n\ge 2)$ là các số không âm có tổng bằng $1$. CMR: $$\sqrt{\dfrac{1-x_1}{1+x_1}}+\sqrt{\dfrac{1-x_2}{1+x_2}}+...+\sqrt{\dfrac{1-x_n}{1+x_n}}\le n-2+\dfrac{2}{\sqrt{3}}$$

Ví dụ 7: (VMEO 2005) Cho $a,b,c$ là các số thực dương cho trước và $x, y, z$ là các số thực dương thỏa mãn $ax+by+cz=xyz$. Tìm GTNN của: $$P=x+y+z$$ Lời giải:
Bổ đề: Với các số thực dương $a, b, c$ cho trước. Khi đó tồn tại duy nhất $d>0$ sao cho: $$\dfrac{1}{a+d}+\dfrac{1}{b+d}+\dfrac{1}{c+d}=\dfrac{2}{d}$$ Chứng minh bổ đề: Viết biểu thức trên lại thành: $$\dfrac{a}{a+d}+\dfrac{b}{b+d}+\dfrac{c}{c+d}=1$$ Xem đây là một hàm $f$ theo $d$. Dễ thấy $f$ liên tục và $f$ nghịch biến. Mặt khác $\lim_{d\to 0}f(d) =3$ và $\lim_{d\to +\infty } f(d)=0$ nên tồn tại duy nhất số $d>0$ thỏa mãn đẳng thức trên. Bổ đề được chứng minh.

Quay lại bài toán. Đặt $x=\sqrt{\dfrac{bc}{a}}m, y=\sqrt{\dfrac{ca}{b}}n, z=\sqrt{\dfrac{ab}{c}}p$. Khi đó ta có: $$m+n+p=mnp$$ Điều này gợi cho ta chọn $A, B, C$ là ba góc của một tam giác nhọn sao cho: $m=\tan A, n=\tan B, p=\tan C$. Bây giờ, ta cần tìm GTNN của: $$P=\sqrt{\dfrac{bc}{a}}\tan A+\sqrt{\dfrac{ca}{b}}\tan B+\sqrt{\dfrac{ca}{b}}\tan C$$ Xét hàm $f(x)=\tan x, x\in \left(0;\dfrac{\pi}{2}\right)$. Ta có : $f"(x)=2\tan x. (\tan ^2 x+1)>0,\forall x\in\left(0, \dfrac{\pi}{2}\right)$ nên $f$ lõm trên $\left(0; \dfrac{\pi}{2}\right)$. Ta chọn $d$ là số thực dương sao cho: $$\dfrac{1}{a+d}+\dfrac{1}{b+d}+\dfrac{1}{c+d}=\dfrac{2}{d}$$ Sự tồn tại của $d$ hoàn toàn được khẳng định theo bổ đề. Áp dụng BDT tiếp tuyến, ta có: $$\begin{aligned} & f(A)\ge f'\left(\arctan\sqrt{\dfrac{a(b+d)(c+d)}{bc(a+d)}}\right)\left[A-\arctan\sqrt{\dfrac{a(b+d)(c+d)}{bc(a+d)}}\right]+f\left(\arctan\sqrt{\dfrac{a(b+d)(c+d)}{bc(a+d)}}\right)\\\Rightarrow &\tan A\ge \dfrac{a(b+d)(c+d)+bc(a+d)}{bc(a+d)}\left[A-\arctan\sqrt{\dfrac{a(b+d)(c+d)}{bc(a+d)}}\right]+\sqrt{\dfrac{a(b+d)(c+d)}{bc(a+d)}}\\&\Rightarrow\sqrt{\dfrac{bc}{a}}\tan A\ge \dfrac{2abc+(ab+bc+ca)d+ad^2}{\sqrt{abc}(a+d)}\left[A-\arctan\sqrt{\dfrac{a(b+d)(c+d)}{bc(a+d)}}\right]+\sqrt{\dfrac{(b+d)(c+d)}{a+d}}\end{aligned}$$ Chú ý rằng vì $$\dfrac{1}{a+d}+\dfrac{1}{b+d}+\dfrac{1}{c+d}=\dfrac{2}{d}$$ nên ta có ngay  $$d^3=2abc+(ab+bc+ca)d$$ Suy ra: $$\sqrt{\dfrac{bc}{a}}\tan A\ge\dfrac{d^2}{\sqrt{abc}}\left[ A-\arctan\sqrt{\dfrac{a(b+d)(c+d)}{bc(a+d)}}\right] +\sqrt{\dfrac{(b+d)(c+d)}{a+d}}$$ Tương tự, ta cũng có: $$\sqrt{\dfrac{ca}{b}}\tan B\ge \dfrac{d^2}{\sqrt{abc}}\left[ B-\arctan\sqrt{\dfrac{b(a+d)(c+d)}{ac(b+d)}}\right] +\sqrt{\dfrac{(a+d)(c+d)}{b+d}}\\\sqrt{\dfrac{ab}{c}}\tan C\ge \dfrac{d^2}{\sqrt{abc}}\left[ C-\arctan\sqrt{\dfrac{c(a+d)(b+d)}{ab(c+d)}}\right] +\sqrt{\dfrac{(a+d)(b+d)}{c+d}}$$ Chú ý rằng: $$\sqrt{\dfrac{a(b+d)(c+d)}{bc(a+d)}}+\sqrt{\dfrac{b(c+d)(a+d)}{ca(b+d)}}+\sqrt{\dfrac{c(a+d)(b+d)}{ab(c+d)}}=\sqrt{\dfrac{(a+d)(b+d)(c+d)}{abc}}$$ nên: $$A+B+C-\arctan\sqrt{\dfrac{a(b+d)(c+d)}{bc(a+d)}}-\arctan\sqrt{\dfrac{b(c+d)(a+d)}{ca(b+d)}}+\arctan\sqrt{\dfrac{c(a+d)(b+d)}{ab(c+d)}}=0$$ Do đó: $$P\ge \sqrt{\dfrac{(a+d)(b+d)}{c+d}}+\sqrt{\dfrac{(b+d)(c+d)}{a+d}}+\sqrt{\dfrac{(c+d)(a+d)}{b+d}}$$ Vậy: $$\min P=\sqrt{\dfrac{(b+d)(c+d)}{a+d}}+\sqrt{\dfrac{(c+d)(a+d)}{b+d}}+\sqrt{\dfrac{(a+d)(b+d)}{c+d}}$$

Kết thúc bài viết, tôi xin đề nghị một số bài tập tự luyện: 
 1. Cho $a, b, c, d\in [0; 1]$. CMR: $$(1-a)(1-b)(1-c)(1-d)+a+b+c+d\ge 1$$
 2. (THTT 2002) Cho $a, b, c$ là các số thực có tổng bình phương bằng $2$. Tìm GTLN của $$P=3(a+b+c)-22abc$$
 3. Cho $a, b, c, d\ge 0$ thỏa $a+b+c+d=4$. CMR: $$16+2abcd\ge 3(ab+ac+ad+bc+bd+cd)$$
 4. (Viet Nam TST 2001, Trần Nam Dũng) Cho $a, b, c$ là các số dương thỏa $21ab+2bc+8ca\le 12$. Tìm GTNN của: $$P=\dfrac{1}{a}+\dfrac{2}{b}+\dfrac{3}{c}$$
 5. (IMO 1984) Cho $x,y,z\ge 0$ thỏa $x+y+z=1$. CMR: $$0\ge xy+yz+zx-2xyz\le \dfrac{7}{27}$$
 6. Cho $x,y,z>0$ thỏa $x+y+z=1$. Tìm GTLN của $$F=\dfrac{x}{x^2+1}+\dfrac{y}{y^2+1}+\dfrac{z}{z^2+1}$$
 7. (VMEO 2004) Cho tam giác nhọn $ABC$. Tìm GTNN của: $$P=\tan A+2\tan B+5\tan C$$
 8. Cho $x, y, z, t\ge 0$ thỏa $x+y+z+t=4$. CMR: $$(1+x)(1+3y)(1+3z)(1+3t)\le 125 +131xyzt$$

Tài liệu tham khảo:
 $[1]$. Bất đẳng thức- Võ Quốc Bá Cẩn.
 $[2]$. Kỹ thuật đánh giá trên biên - Phạm Quốc Thắng.
 $[3]$. Nguyên lý biên - Cao Đình Huy.
 $[4]$. www.diendantoanhoc.net

Monday, July 13, 2015

ĐỀ THI IMO 2015



IMO 2015
Ngày 10-07-2015

Bài 1. Cho $S$ là một tập hữu hạn các điểm trên mặt phẳng. Tập $S$ được gọi là "cân bằng" nếu với với hai điểm $A,B$ phân biệt bất kì thuộc $S$ thì luôn tồn tại điểm $C$ thuộc $S$ thoả mãn $AC=BC$. Tập $S$ được gọi là "không tâm" nếu với bất kì ba điểm phân biệt $A,B,C$ thuộc $S$ thì không có điểm $P$ nào thoả mãn $PA=PB=PC$.
     a) Chứng minh rằng với mọi số nguyên $n \ge 3$, tồn tại một tập "cân bằng" với $n$ điểm.
     b) Tìm tất cả số nguyên $n \ge 3$, sao cho tồn tại một tập "cân bằng" và "không tâm" cho $n$ điểm.

Bài 2. Tìm mọi số nguyên dương $(a,b,c)$ thoả mãn $ab-c,bc-a,ca-b$ đều là luỹ thừa của $2$.

Bài 3. Cho tam giác nhọn $ABC$, $AB>AC$ có đường tròn ngoại tiếp $\Gamma$, trực tâm $H$ và chân đường cao $F$ hạ từ $A$. $M$ là trung điểm $BC$. $Q$ là điểm trên $\Gamma$ thoả mãn $\angle HQA= 90^{\circ}$, và $K$ là điểm trên $\Gamma$ sao cho $\angle HKQ=90^{\circ}$. $A,B,C,K,Q$ là các điểm phân biệt, và chúng nằm trên $\Gamma$ theo đúng thứ tự đó. Chứng minh rằng đường tròn ngoai tiếp hai tam giác $KQH$ và $FKM$ tiếp xúc với nhau.


IMO 2015
Ngày 11-07-2015


Bài 4. Tam giác $ABC$ nội tiếp đường tròn $\Omega$ có tâm $O$. Một đường tròn $\Gamma$ với tâm $A$ cắt cạnh $BC$ tại $D$ và $E$ sao cho $B,D,E,C$ phân biệt và nằm trên đường thẳng $BC$ theo đúng thứ tự này. $F,G$ là giao điểm của $\Omega$ và $\Gamma$ sao cho $A,F,B,C,G$ nằm trên $\Omega$ theo đúng thứ tự này. $K$ là giao điểm thứ hai của đường tròn ngoại tiếp $\triangle BDF$ và cạnh $AB$. $L$ là giao điểm thứ hai của đường tròn ngoại tiếp $\triangle CGE$ và cạnh $CA$.

Giả sử đường thẳng $FK$ và $GL$ phân biệt và cắt nhau tại $X$. Chứng minh rằng $X$ nằm trên đường thẳng $AO$.

Bài 5. Kí hiệu $\mathbb{R}$ là tập các số thực. Xác định tất cả các hàm số $f: \mathbb{R} \to \mathbb{R}$ thoả mãn $$f(x+f(x+y))+f(xy)=x+f(x+y)+yf(x)$$ với mọi số thực $x,y$.

Bài 6. Cho dãy $a_1,a_2, \cdots $ các số nguyên thoả mãn điều kiện sau:
     (i) $1 \le a_j \le 2015$ với mọi $j \ge 1$;
     (ii) $k+a_k \ne l+a_l$ với mọi $1 \le k<l$.

Chứng minh rằng tồn tại hai số nguyên dương $b$ và $N$ thoả mãn $$\left | \sum_{j=m+1}^n(a_j-b) \right | \le 1007^2$$ với mọi số nguyên $m,n$ thoả mãn $n>m \ge N$.

Nguồn: diendantoanhoc.net

Sunday, July 12, 2015

Bài toán [Nguyễn Văn Huyện]: Cho $a,b,c$ là các thực thỏa $a+b, b+c, c+a$ là các số dương. Chứng minh rằng: $$9(a+b)(b+c)(c+a)\ge 8(a+b+c)(ab+bc+ca)$$
Lời giải:
Do $\min\{a+b; b+c; c+a\}>0$ nên trong ba số $a, b, c$ phải có ít nhất hai số không âm. Giả sử đó là $b, c$. Bất đẳng thức cần chứng minh tương đương với: $$(a+b+c)(ab+bc+ca)\ge 3abc\\\Leftrightarrow (b+c)a^2+[(b+c)^2-8bc]a+bc(b+c)\ge 0$$ Vì $b+c>0$, xét biệt thức $$\Delta =[(b+c)^2-8bc]^2-4bc(b+c)^2=(b+c)^4-20bc(b+c)^2+64b^2c^2$$
Ta sẽ CM $\Delta \le 0$, tức là: $$(b+c)^4-20bc(b+c)^2+64b^2c^2\le 0$$ Chuẩn hóa cho $b+c=1\Rightarrow bc\le \dfrac{1}{4}$. Khi đó ta cần chứng minh: $$64b^2c^2-20bc+1\le 0$$ hiển nhiên đúng với $0<bc\le \dfrac{1}{4}$. Bài toán được giải quyết.

Sunday, June 21, 2015

BDT đề 07/50 - Đặng Thành Nam

Bài toán: Cho $x,y,z$ là các số thực thỏa mãn $x+y+z=0$. Tìm GTNN của $$P=3^{|\cos x|}+3^{|\cos y|}+3^{|\cos z|}-3\max\{|\cos x|, |\cos y|, |\cos z|\}$$
Lời giải:
Không giảm tính tổng quát, giả sử $|\cos z|=\max\{|\cos x|, |\cos y|, |\cos z|\}$
Sử dụng BDT Bernoulli, ta có: $$3^{|\cos x|}\ge 1+|\cos x|$$ Suy ra: $$P\ge 2+|\cos x|+|\cos y|+3^{|\cos z|}-3|\cos z|$$ 
Bổ đề [Ukraine 2006] Cho $x,y$ là các số thực. Chứng minh rằng: $$|\cos x|+|\cos y|+|\cos (x+y)|\ge 1$$
Chứng minh bổ đề: 
 Đặt $a=|\cos x|, b=|\cos y|$. Sử dụng BDT quen thuộc $|a-b|\ge ||a|-|b||$. Ta có: $$|\cos (x+y)=|\cos x\cos y-\sin x\sin y|\ge ||\cos x\cos y|-|\sin x\sin y||=|ab-\sqrt{(1-a^2)(1-b^2)}|$$ Theo đó ta cần chứng minh: $$a+b+|\sqrt{(1-a^2)(1-b^2)}-ab|\ge 1$$ Ta xét khi $a+b\le 1$. Vì $ab>0$ nên: $$a^2+b^2\le (a+b)^2\le 1$$ Suy ra: $$(1-a^2)(1-b^2)-a^2b^2=1-a^2-b^2+a^2b^2-a^2b^2=1-a^2-b^2\ge 0$$ Hay nói cách khác: $$|\sqrt{(1-a^2)(1-b^2)}-ab|=\sqrt{(1-a^2)(1-b^2)}-ab$$ Bổ đề cần chứng minh trở thành: $$a+b+\sqrt{(1-a^2)(1-b^2)}-ab\ge 1\\\Leftrightarrow \sqrt{(1-a^2)(1-b^2)}\ge (1-a)(1-b),\text{ Hiển nhiên vì } 0\le a,b\le 1$$ Bổ đề được chứng minh.

Quay lại bài toán, áp dụng bổ đề trên ta có: $$P\ge 3+3^{|\cos z|}-4|\cos z|$$ Xét hàm số $f(t)=3+3^t-4t$ với $0\le t\le 1$ ta tính được: $$f'(t)=3^t\ln 3-4<0,\forall t\in [0,1]$$ Suy ra $f$ nghịch biến. Suy ra: $$P\ge f(1)=2$$ Dấu đẳng thức xảy ra khi: $\begin{cases}|\cos x|=|\cos y|\\x+y+z=0\\|\cos z|=1\end{cases}$ và các hoán vị. 
Kết luận: $$\boxed{\min P=2}$$

Saturday, June 6, 2015

Bài toán (Đề KT đội tuyển HSG trường chuyên Long An) 
 Cho $p$ là một số nguyên tố lẻ và $a, b$ là hai số tự nhiên sao cho $a+b$ chia hết cho $p$ và $a-b$ chia hết cho $p-1$. Chứng minh rằng: $a^b+b^a$ chia hết cho $2p$
Lời giải:
 Không giảm tính tổng quát, giả sử: $a\ge b$. Gọi $r$ là số dư trong phép chia $a$ cho $p$, khi đó: $a\equiv r\pmod{p}$ Vì $p\;|\; a+b$ nên suy ra $b\equiv -r\pmod{p}$. Do đó: $$\begin{aligned} & a^b+b^a\equiv r^b-r^a\pmod{p}\\\Leftrightarrow & a^b+b^a\equiv r^b(1-r^{a-b})\pmod{p}\end{aligned}$$ Mặt khác, do $p-1\;|\; a-b$ nên tồn tại $k\in\mathbb{N}$ sao cho $a-b=k(p-1)$.
 Theo định lý Fermat, ta có: $$\begin{aligned}& r^{p-1}\equiv 1\pmod{p}\\\Rightarrow & r^{k(p-1)}\equiv 1\pmod{p}\\\Rightarrow &r^{a-b}\equiv 1\pmod{p}\end{aligned}$$ Do đó: $$a^b+b^a\equiv 0\pmod{p}$$
Mặt khác vì $a,b$ là các số tự nhiên lẻ nên $a^b+b^a\equiv 0\pmod{2}$. Từ đó suy ra đpcm.

Wednesday, May 20, 2015

Phương trình hàm trong đề KT đội tuyển Chuyên Long An

Bài toán: (Đề kiểm tra đội tuyển HSG trường Chuyên Long An)
Tìm tất cả các hàm số $f:\mathbb{R}\to\mathbb{R}$ thỏa mãn: $$f(x^2)=f(y).f(2x-y)+(y-x)^2,\forall x, y \in\mathbb{R}\;\;\;\;\;\;\;\;\;\;\;\;\;\;\;\;\;\;\;\;(*)$$ Lời giải: Trong $(*)$ cho $x=y$ ta được: $$f(x^2)=f^2(x),\forall x\in\mathbb{R}\;\;\;\;\;\;\;\;\;\;\;\;\;\;\;\;(1)$$ Trong $(1)$ lấy $x=0$ ta được $f(0)=0\vee f(0)=1$.
Trường hợp 1: $f(0)=0$, trong $(*)$ cho $y=0$ ta được: $$\begin{aligned}&\;\;\;\;f(x^2)=x^2\\&\Leftrightarrow f^2(x)=x^2, \forall x\in\mathbb{R}\end{aligned}$$
Từ đây suy ra $f(x)=x$ hoặc $f(x)=-x$. Thử lại ta thấy chỉ có hàm số $f(x)=x,\forall x\in\mathbb{R}$ thỏa mãn bài toán.
Trường hợp 2: $f(0)=1$, trong $(*)$ cho $y=0$, ta được: $$f(x^2)=f(2x)+x^2,\forall x\in\mathbb{R}\;\;\;\;\;\;\;\;\;\;\;\;\;\;\;(2)$$ Trong $(*)$ thay $y$ bởi $x-y$ ta được: $$f(x^2)=f(x-y)f(x+y)+y^2,\forall x,y\in\mathbb{R}\;\;\;\;\;\;\;\;\;\;\;\;\;(3)$$ Kết hợp $(2)$ và $(3)$ ta được: $$f(2x)+x^2=f(x+y).f(x-y)+y^2,\forall x, y\in\mathbb{R}$$ Đặt $a=x+y,\;b=x-y, \;a,b\in\mathbb{R}$ ta được: $$f(a+b)+ab=f(a).f(b),\forall a,b\in\mathbb{R}\;\;\;\;\;\;\;\;\;\;\;\;\;\;\;\;\;\;(4)$$ Trong $(*)$, ta thay $y$ bởi $b$, $2x-y$ bởi $a$, ta được: $$f\left(\dfrac{(a+b)^2}{4}\right)=f(a).f(b)-\dfrac{(a-b)^2}{4},\forall a, b\in\mathbb{R}\;\;\;\;\;\;\;\;\;\;\;\;\;\;\;\;\;(5)$$ Kết hợp $(4)$ và $(5)$ ta được:  $$\begin{aligned}&\;\;\;f(a+b)+ab=f\left(\dfrac{(a+b)^2}{4}\right)-\dfrac{(a-b)^2}{4},\forall a, b\in\mathbb{R}\\&\Leftrightarrow 4f(a+b)=4f\left(\dfrac{(a+b)^2}{4}\right)-(a+b)^2,\forall a,b\in\mathbb{R}\;\;\;\;\;\;\;\;\;(6)\end{aligned}$$ Trong $(6)$ cho $a+b=4$ ta được $-4^2=0$, vô lý.
Vậy chỉ có hàm số thỏa mãn bài toán: $$f(x)=x,\forall x\in\mathbb{R}$$

Sunday, May 3, 2015

Using S-S method in prove inequality

Bài toán: Cho $x, y, z$ là các số thực không âm thỏa mãn $2x+3y+z=1$. Tìm GTNN của: $$P=4x^2+9y^2+z^2+24xyz$$ Lời giải:
 Thực hiện đổi biến $\left(x,y,z\right)\rightarrow \left(\dfrac{x}{2},\dfrac{y}{3}, z\right)$. Khi đó ta có $x+y+z=1$ đồng thời: $$P=x^2+y^2+z^2+4xyz$$ Chú ý rằng $x+y+z=1$ nên ta có: $$\begin{aligned}P&=\dfrac{x^2+y^2+z^2}{(x+y+z)^2}+\dfrac{4xyz}{(x+y+z)^3}=\dfrac{x^2+y^2+z^2}{(x+y+z)^2}-\dfrac{1}{3}+\dfrac{4xyz}{(x+y+z)^3}-\dfrac{4}{27}+\dfrac{13}{27}\\&=\dfrac{3(x^2+y^2+z^2)-(x+y+z)^2}{3(x+y+z)^2}-4.\dfrac{(x+y+z)^3-27xyz}{27(x+y+z)^3}+\dfrac{13}{27}\\&=\dfrac{2[(x-y)^2+(x-z)(y-z)]}{3(x+y+z)^2}-4.\dfrac{(x+y+z)[(x-y)^2+(x-z)(y-z)]+3[(2z(x-y)^2+(x+y)(x-z)(y-z)]}{27(x+y+z)^3}+\dfrac{13}{27}\\&=M(x-y)^2+N(x-z)(y-z)+\dfrac{13}{27}\end{aligned}$$
 Trong đó: $$\begin{aligned} M&=\dfrac{2}{3(x+y+z)^2}-\dfrac{4(x+y+z+6z)}{27(x+y+z)^3}\\&=\dfrac{18(x+y+z)-4(x+y+z)-24z}{27(x+y+z)^3}=\dfrac{14-24z}{27}\\\\N&=\dfrac{18(x+y+z)-4(x+y+z+3x+3y)}{27(x+y+z)^3}\\&=\dfrac{14(x+y+z)-12(x+y)}{27(x+y+z)^3}=\dfrac{2+12z}{27}\end{aligned}$$ Ngoài ra, chú ý rằng sau khi thực hiện phép đổi biến thì vai trò $x,y,z$ là như nhau nên ta hoàn toàn có thể giải sử $z=\min\{x,y,z\}$ do đó $M, N>0$. Từ đó suy ra: $$P=M(x-y)^2+N(x-z)(y-z)+\dfrac{13}{27}\ge \dfrac{13}{27}$$ Dấu ''='' xảy ra khi và chỉ khi $\begin{cases}x+y+z=1\\x=y=z\end{cases}\text{ hay}\begin{cases}2x+3y+z=1\\2x=3y=z=\dfrac{1}{3}\end{cases} \Leftrightarrow (x,y,z)=\left(\dfrac{1}{6},\dfrac{1}{9},\dfrac{1}{3}\right)$ 
Kết luận: $$\boxed{\min P=\dfrac{13}{27}}$$

Tuesday, April 7, 2015

XUNG QUANH ĐẠI LƯỢNG $A-B, B-C, C-A$

XUNG QUANH ĐẠI LƯỢNG $A-B, B-C, C-A$
 
     Nhận xét chung: Khi đứng trước các bài toán bất đẳng thức ở dạng hoán vị hoặc đối xứng, một ý tưởng hiển nhiên là ta sẽ liên tưởng tới nhận xét: "Mọi đa thức đối xứng đều có thể biểu diễn dưới dạng đối xứng cơ sở". Cụ thể là với đa thức hoán vị vòng quanh $f(a,b,c)$ thì ta có: $$f(a,b,c)=\dfrac{1}{2}\left[ f(a,b,c)+f(c,b,a)\right] +\dfrac{1}{2}\left[ f(a, b, c)-f(c, b, a)\right]$$ Khi đó đại lượng $f(a,b,c)-f(c,b,a)$ có thể phân tích thành các đại lượng theo $a-b, b-c, c-a$. Đồng thời, khi vai trò các biến như nhau, ta hoàn toàn có thể giả sử $a=\min\{a,b,c\}$ hoặc $a=\max\{a, b, c\}$ hoặc $a\ge b\ge c$. Từ đó tìm hướng tiếp cận cụ thể cho bài toán.
 
Bài toán 1:(Trần Nam Dũng, VMO 2008) Cho $a, b, c$ là các số thực không âm đôi một phân biệt. Chứng minh rằng: $$\dfrac{1}{(b-c)^2}+\dfrac{1}{(c-a)^2}+\dfrac{1}{(a-b)^2}\ge\dfrac{4}{ab+bc+ca}$$
  Lời giải: Không giảm tính tổng quát, ta có thể giả sử $c=\min\{a,b,c\}$. Ta nhận thấy: $$(a-c)^2+(b-c)^2=(a-b)^2+2(a-c)(b-c)$$ Suy ra: $$\begin{aligned}\dfrac{1}{(b-c)^2}+\dfrac{1}{(c-a)^2}+\dfrac{1}{(a-b)^2}&=\dfrac{1}{(a-b)^2}+\dfrac{(a-c)^2+(b-c)^2}{(a-b)^2(b-c)^2}\\&=\dfrac{1}{(a-b)^2}+\dfrac{(a-b)^2}{(b-c)^2(a-c)^2}+\dfrac{2}{(a-c)(b-c)}\\&\ge\dfrac{2}{(a-c)(b-c)}+\dfrac{2}{(a-c)(b-c)}=\dfrac{4}{(a-c)(b-c)}\end{aligned}$$ Mặt khác, do $c=\min\{a, b, c\}$ nên: $$c(2a+2b-c)\ge 0\Rightarrow \dfrac{4}{(a-c)(b-c)}\ge\dfrac{4}{ab+bc+ca}$$ Từ đó ta có đpcm.
 
Cách khác:
Không mất tính tổng quát, giả sử $a>b>c\ge 0$. Đặt $x=a-b, y=b-c$ Khi đó, $x, y>0$ và: $$ab+bc+ca\ge ab=(c+y)(c+x+y)\ge y(x+y)$$ Theo đó, ta lui về chứng minh một kết quả mạnh hơn: $$\begin{aligned} &\;\;\;\;\;\;\dfrac{1}{x^2}+\dfrac{1}{y^2}+\dfrac{1}{(x+y)^2}\ge\dfrac{4}{y(x+y)}\\&\Leftrightarrow y(x+y)\left[\dfrac{1}{x^2}+\dfrac{1}{y^2}+\dfrac{1}{(x+y)^2}\right]\ge 4\\&\Leftrightarrow \dfrac{y}{x}\left( 1+\dfrac{y}{x}\right)+\dfrac{x}{y}+\dfrac{y}{x+y}\ge 3\end{aligned}$$ Đặt $t=\dfrac{x}{y}$, lúc này ta cần chứng minh: $$\dfrac{t+1}{t^2}+t+\dfrac{1}{t+1}\ge 3$$ Bất đẳng thức này tương đương với: $$\dfrac{(t^2-t-1)^2}{t^2(t+1)}\ge 0$$ Đây là một kết quả hiển nhiên. Bài toán được chứng minh

Bài toán 2: Cho $a, b, c$ là các số thực phân biệt. Chứng minh rằng: $$\left[(a-b)^4+(b-c)^4+(c-a)^4\right]\left[\dfrac{1}{(a-b)^4}+\dfrac{1}{(b-c)^4}+\dfrac{1}{(c-a)^4}\right]\ge\dfrac{297}{8}$$
  Lời giải:
Không giảm tính tổng quát, ta có thể giả sử $a\ge b\ge c$. Đặt $x=a-b, y=b-c$. Khi đó ta có $x, y\ge 0$ và $c-a=-x-y$. Bất đẳng thức cần chứng minh tương đương với: $$\left[ x^4+y^4+(x+y)^4\right]\left[\dfrac{1}{x^4}+\dfrac{1}{y^4}+\dfrac{1}{(x+y)^4}\right]\ge\dfrac{297}{8}$$ Tới đây, ta có đánh giá: $$x^4+y^4\ge\dfrac{(x+y)^4}{8}$$ và $$\dfrac{1}{x^4}+\dfrac{1}{y^4}\ge\dfrac{32}{(x+y)^4}$$ Suy ra: $$\left[ x^4+y^4+(x+y)^4\right]\left[\dfrac{1}{x^4}+\dfrac{1}{y^4}+\dfrac{1}{(x+y)^4}\right]\ge\left[\dfrac{(x+y)^4}{8}+(x+y)^4\right]\left[\dfrac{32}{(x+y)^4}+\dfrac{1}{(x+y)^4}\right]=\dfrac{297}{8}$$ Phép chứng minh hoàn tất.
 
  Bài toán 3: (Trần Quốc Anh) Cho $a, b, c$ là các số thực không âm thỏa mãn $a+b+c=3$. Tìm giá trị lớn nhất của biểu thức: $$P=(a^3+b^3+c^3)(a^2-b^2)(b^2-c^2)(c^2-a^2)$$
  Lời giải:
  Do cần tìm giá trị lớn nhất của $P$ nên ta chỉ cần xét trong trường hợp $a\le b\le c$ là được. Áp dụng bất đẳng thức AM-GM, ta có đánh giá: $$\begin{aligned}(a+b+c)^3&=a^3+b^3+c^3+3(a+b)(b+c)(c+a)\\&\ge 2\sqrt{3(a^3+b^3+c^3)(a+b)(b+c)(c+a)}\end{aligned}$$ Suy ra: $$(a^3+b^3+c^3)(a+b)(b+c)(c+a)\le\dfrac{243}{4}$$ Do $a\le b\le c$ nên $$(a-b)^2\le b^2, (c-a)^2\le c^2$$. Kéo theo: $$\begin{aligned}(a-b)^2(b-c)^2(c-a)^2&\le b^2c^2(b-c)^2=\dfrac{1}{4}.2bc.2bc.(b^2-2bc+c^2)\\&\le \dfrac{1}{4}\left(\dfrac{2bc+2bc+b^2-2bc+c^2}{3}\right)^3=\dfrac{(b+c)^6}{108}\le\dfrac{(a+b+c)^6}{108}=\dfrac{27}{4}\end{aligned}$$ Suy ra: $$(a-b)(b-c)(c-a)\le \dfrac{3\sqrt{3}}{2}$$ Từ đó, ta có ngay: $$P=(a^3+b^3+c^3)(a^2-b^2)(b^2-c^2)(c^2-a^2)\le\dfrac{729\sqrt{3}}{8}$$ Dấu đẳng thức xảy ra khi và chỉ khi $(a,b,c)=(0,3-\sqrt{3},3+\sqrt{3})$ và các hoán vị tương ứng. Kết luận $\max P=\dfrac{729\sqrt{3}}{8}$.
 
  Bài toán 4: (Phan Thành Nam) Cho $a, b, c$ là các số thực không âm. Chứng minh rằng: $$(a^2+b^2+c^2)^2\ge 4(a+b+c)(a-b)(b-c)(c-a)$$
  Lời giải:
  Ta chỉ cần xét trong trường hợp $a\le b\le c$ là đủ. Sử dụng bất đẳng thức AM-GM, ta có: $$\begin{aligned}4(a+b+c)(a-b)(b-c)(c-a)&=4(a+b+c)(b-a).(c-b)(c-a)\\&\le\left[(a+b+c)(b-a)+(c-b)(c-a)\right]^2\\&=\left[b^2+c^2+(b-2c)a-a^2\right]^2\\&\le (b^2+c^2)^2\le (a^2+b^2+c^2)^2\end{aligned}$$ Phép chứng minh hoàn tất.
 
  Bài toán 5: Cho $a, b, c$ là các số thực thỏa $a^2+b^2+c^2=1$. Tìm giá trị lớn nhất và nhỏ nhất của: $$P=(a+b+c)(a-b)(b-c)(c-a)$$
  Lời giải:
  Xét $$P^2=(a+b+c)^2(a-b)^2(b-c)^2(c-a)^2$$ Đặt $p=a+b+c, q=ab+bc+ca, r=abc$. Khi đó ta có: $$(a-b)^2(b-c)^2(c-a)^2=-27r^2+2(9pq-2p^3)r+p^2q^2-4q^3$$ Suy ra: $$P^2=(1+2q)(-27r^2+2(9pq-2p^3)r+p^2q^2-4q^3)$$ Xem $P^2$ như là một tam thức bậc hai ẩn $r$. Ta tính được: $$\Delta '=(1+2q)^2\left[(9pq-2p^3)^2+27(p^2q^2-4q^3)\right]$$ Chú ý rằng $p^2-2q=1$ và sau một vài bước biến đổi cơ bản ta dễ dàng suy ra được: $$\Delta '=4(1+2q)^2(1-q)^3$$ Từ đó suy ra, $$P^2\le\dfrac{4(1+2q)^2(1-q)^3}{27}$$ Thực hiện khảo sát hàm số $$f(q)=\dfrac{4(1+2q)^2(1-q)^3}{27}$$ ta dễ dàng suy ra: $$f(q)=\dfrac{4(1+2q)^2(1-q)^3}{27}\le f\left(-\dfrac{1}{8}\right)=\dfrac{81}{512}$$. Từ đó suy ra: $$P^2\le\dfrac{81}{512}$$ hay $$-\dfrac{9\sqrt{2}}{32}\le P\le \dfrac{9\sqrt{2}}{32}$$ Từ đó suy ra:
  + $\min P=-\dfrac{9\sqrt{2}}{32}$ đạt được khi và chỉ khi $(a, b, c)=\left(\dfrac{3\sqrt{6}+2\sqrt{3}}{12}, \dfrac{\sqrt{3}}{6}, \dfrac{2\sqrt{3}-3\sqrt{6}}{12}\right)$ và các hoán vị tương ứng.
  + $\max P=\dfrac{9\sqrt{2}}{32}$ đạt được khi và chỉ khi $(a, b, c)=\left(\dfrac{2\sqrt{3}-3\sqrt{6}}{12}, \dfrac{\sqrt{3}}{6},\dfrac{3\sqrt{6}+2\sqrt{3}}{12}\right)$ và các hoán vị tương ứng.

   Nếu ta gọi $M$ là giá trị lớn nhất (hay nhỏ nhất) của $P$. Sau khi thuần nhất hóa, ta cần định giá trị $M$ nhỏ nhất sao cho bất đẳng thức sau đúng: $(a-b)^2(b-c)^2(c-a)^2(a+b+c)^2\ge M^2(a^2+b^2+c^2)^4$ Dĩ nhiên là lúc này ta bỏ qua điều kiện $a^2+b^2+c^2=1$. Tiếp tục biến đổi ta thu được bài toán của kì thi IMO 2006:
Bài toán:(IMO 2006) Tìm số thực $M$ nhỏ nhất sao cho với mọi $a, b, c$ thực, ta có bất đẳng thức: $$|ab(a^2-b^2)+bc(b^2-c^2)+ca(c^2-a^2)|\le M(a^2+b^2+c^2)^2$$

  Bài toán 6: Cho $a, b, c$ là các số không âm có tổng bằng 1. Tìm giá trị lớn nhất của: $$P=(a-b)^3+(b-c)^3+(c-a)^3$$
  Lời giải:
  Ta chú ý thấy $a-b+b-c+c-a=0$ nên dễ dàng suy ra: $$(a-b)^3+(b-c)^3+(c-a)^3=3(a-b)(b-c)(c-a)$$ Theo đó, ta cần tìm giá trị lớn nhất của $$Q=(a-b)(b-c)(c-a)$$ Không mất tính tổng quát, giả sử $a\le b\le c$. Sử dụng bất đẳng thức AM-GM, ta có: $$\begin{aligned}Q&\le\dfrac{1}{2}\left[\dfrac{(\sqrt{3}+1)(b-a)+(\sqrt{3}-1)(c-a)+(c-b)}{3}\right]^3\\&=\dfrac{1}{2}\left(\dfrac{b+c-2a}{\sqrt{3}}\right)^3=\dfrac{1}{2}\left(\dfrac{1-3a}{\sqrt{3}}\right)^3\le\dfrac{1}{2.(\sqrt{3})^3}=\dfrac{\sqrt{3}}{18}\end{aligned}$$ Suy ra: $$P=3Q\le\dfrac{\sqrt{3}}{6}$$ Dấu đẳng thức xảy ra khi $(a,b,c)=\left(0,\dfrac{3-\sqrt{3}}{6}, \dfrac{3+\sqrt{3}}{6}\right)$ và các hoán vị tương ứng. Kết luận: $\max P=\dfrac{\sqrt{3}}{6}$
 
  Bài toán 7: Cho $a, b, c$ là các số thực phân biệt có tổng bằng $1$ và thỏa $ab+bc+ca>0$. Tìm giá trị nhỏ nhất của: $$P=\dfrac{2}{|a-b|}+\dfrac{2}{|b-c|}+\dfrac{2}{|c-a|}+\dfrac{5}{\sqrt{ab+bc+ca}}$$
  Lời giải:
  Không mất tính tổng quát, giả sử $a<b<c$. Khi đó, ta có: $$P=\dfrac{2}{b-a}+\dfrac{2}{c-b}+\dfrac{2}{c-a}+\dfrac{5}{\sqrt{ab+bc+ca}}$$ Áp dụng bất đẳng thức Cauchy Schwarz, ta có: $$P\ge \dfrac{10}{c-a}+\dfrac{5}{\sqrt{ab+bc+ca}}$$ Ngoài ra, theo AM-GM ta còn có: $$\begin{aligned} 3(ab+bc+ca)-1&=3(ab+bc+ca)-(a+b+c)^2\\&=ab+bc+ca-a^2-b^2-c^2\\&=-(c-a)^2+(c-b)(b-a)\\&\le -(c-a)^2+ \dfrac{(c-b+b-a)^2}{4} =-\dfrac{3(c-a)^2}{4}\end{aligned}$$ Suy ra: $$ab+bc+ca\le \dfrac{4-3(c-a)^2}{12}$$ Từ đó suy ra: $$P\ge \dfrac{10}{c-a}+\dfrac{10\sqrt{3}}{\sqrt{4-3(c-a)^2}}$$ Đặt $x=c-a, x>0$ thế thì:$$P\ge \dfrac{10}{x}+\dfrac{10\sqrt{3}}{\sqrt{4-3x^2}}$$ Tới đây, bằng việc khảo sát hàm số $$f(x)=\dfrac{10}{x}+\dfrac{10\sqrt{3}}{\sqrt{4-3x^2}}$$ ta dễ dàng nhận thấy $$f(x)\ge 10\sqrt{6}$$. Cho nên ta có ngay: $$P\ge 10\sqrt{6}$$ Dấu đẳng thức xảy ra khi: $\begin{cases}a+b+c=1\\b-a=c-b\\c-a=\sqrt{\dfrac{2}{3}}\\ab+bc+ca>0\end{cases}\Leftrightarrow \begin{cases}a=\dfrac{1}{3}-\dfrac{1}{\sqrt{6}}\\\\b=\dfrac{1}{3}\\\\c=\dfrac{1}{3}+\dfrac{1}{\sqrt{6}}\end{cases}$
Kết luận, $\min P=10\sqrt{6}$ đạt được khi $(a,b,c)=\left(\dfrac{1}{3}-\dfrac{1}{\sqrt{6}},\dfrac{1}{3},\dfrac{1}{3}+\dfrac{1}{\sqrt{6}}\right)$ và các hoán vị tương ứng.
 
   Cuối cùng, kết thúc bài viết tôi xin đề nghị một số bài tập tự luyện:
   1. (Trần Quốc Anh) Cho $a, b, c$ là các số thực không âm. Chứng minh rằng: $$(a^2+b^2+c^2)^3\ge 27(a-b)^2(b-c)^2(c-a)^2$$
  2. Cho $a, b, c$ là các số thực dương. Chứng minh rằng: $$\sqrt[3]{(a+b)^2(b+c)^2(c+a)^2}\ge\sqrt[3]{(a-b)^2(b-c)^2(c-a)^2}+4\sqrt[3]{a^2b^2c^2}$$
  3. (IMO 1983) Cho $a, b, c$ là độ dài ba cạnh của một tam giác. Chứng minh rằng: $$a^2b(a-b)+b^2c(b-c)+c^2a(c-a)\ge 0$$
  4.(Nguyễn Văn Thạch) Cho các số thực dương $a, b, c$ đôi một phân biệt. Chứng minh rằng: $$\dfrac{1+a^2b^2}{(a-b)^2}+\dfrac{1+b^2c^2}{(b-c)^2}+\dfrac{1+c^2a^2}{(c-a)^2}\ge\dfrac{3}{2}$$
  5. (Võ Quốc Bá Cẩn) Cho $a, b, c$ là các số thực không âm có tổng bình phương bằng $1$. Tìm giá trị lớn nhất của: $$P=(a-b)(b-c)(c-a)(a+b+c)$$
Tài liệu tham khảo: 
$\left[ 1\right]$. Vẻ đẹp bất đẳng thức qua các kỳ thi Olympic - Trần Phương (chủ biên).
$\left[ 2\right]$. Bất đẳng thức hiện đại - Võ Quốc Bá Cẩn.
$\left[ 3\right]$. Một số tài liệu từ Internet.

Saturday, April 4, 2015

Bài toán: (Olympic 30.04 lớp 11 năm 2014-2015) Tìm tất cả đơn ánh $f:\mathbb{R}\to\mathbb{R}$ thỏa: $$f(x^3)+f(y^3)=(x+y)\left[f^2(x)-f(x)f(y)+f^2(y)\right] ,\; \forall x,y\in\mathbb{R}\;\;\;\;\;\;\;\;\;\;\;\;\;\;\;\;\;\; (1)$$ Lời giải: 
Trong $(1)$ cho $x=y$ ta được: $$f(x^3)=xf^2(x),\;\forall x\in\mathbb{R}\;\;\;\;\;\;\;\;\;\;\;\;\;\;\;\;\;\;(2)$$ Trong $(2)$ cho $x=0$, ta được: $f(0)=0$
Trong $(2)$ cho $x=1$ ta được $f(1)=0\;\vee f(1)=1$.

Trường hợp 1: $f(1)=0$. Trong $(1)$ cho $y=1$ ta được: $$f(x^3)=(x+1)f^2(x),\forall x\in\mathbb{R}\;\;\;\;\;\;\;\;\;\;\;\;\;\;\;\;(3)$$ Kết hợp với: $$f(x^3)=xf^2(x),\forall x\in\mathbb{R}\;\;\;\;\;\;\;\;\;\;\;\;\;\;\;\;\;(4)$$ Từ $(3)$ và $(4)$ suy ra: $f(x)\equiv 0, \forall x\in\mathbb{R}$ Tuy nhiên vì $f$ đơn ánh nên ta loại nghiệm này.

Trường hợp 2: $f(1)=1$. Trong $(1)$ cho $y=1$ ta được: $$f(x^3)+1=(x+1)\left[ f^2(x)-f(x)+1\right],\;\forall x\in\mathbb{R}$$ Kết hợp với $$f(x^3)=xf^2(x),\forall x\in\mathbb{R}$$ Suy ra: $$\begin{aligned}&\;\;\;\;\;\;  xf^2(x)+1=(x+1)\left[ f^2(x)-f(x)+1\right]\\& \Leftrightarrow f^2(x)-xf(x)-f(x)+x=0\\& \Leftrightarrow \left[ f(x)-1\right]\left[f(x)-x\right] =0\end{aligned}$$ Tuy nhiên vì $f$ đơn ánh nên $f(x)=1\Leftrightarrow x=1$, tức là ta thu được $f(x)=x,\forall x\in\mathbb{R}$. Kết luận: $$ f(x)\equiv x.\;\forall x\in\mathbb{R}$$

Wednesday, April 1, 2015

Bài toán : Tìm hàm $f:\mathbb{R}\to\mathbb{R}$ liên tục trên $\mathbb{R}$ và thỏa: $$f(xy+x+y)=f(xy)+f(x)+f(y),\;\forall x,y\in\mathbb{R}\;\;\;\;\;\;\;\;\;\;\;\;\;\; (1)$$ Lời giải:
Trong $(1)$ cho $x=y=0$ ta được $f(0)=0$.
Tiếp tục cho $y=-1$ ta được: $$f(-x)=-f(x),\forall x\in\mathbb{R}$$ Như vậy $f$ là hàm lẻ. Cho $y=1$ từ $(1)$ ta được: $$f(2x+1)=2f(x)+1,\forall x\in\mathbb{R}\;\;\;\;\;\;\;\;\;\;\;\;\;\;(2)$$ Trong $(2)$ thay $x$ bởi $xy+x+y$ ta được $$\begin{aligned}f\left( 2(xy+x+y)+1\right) &=2f(xy+x+y)+f(1)\\&= 2f(xy)+2f(x)+2f(y)+f(1) ,\,\forall x,y\in\mathbb{R}\end{aligned}$$ Ngoài ra ta còn có: $$\begin{aligned}f\left( 2(xy+x+y)+1\right) &=f\left( 2x+1+y(2x+1)+y\right)\\&= f(2x+1)+f(y)+f\left( (2x+1)y\right)\\&= 2f(x)+f(1)+f(y)+f(2xy+y), \;\forall x, y\in\mathbb{R}\end{aligned}$$ Từ đó suy ra: $$f(2xy+y)=2f(xy)+f(y),\;\forall x,y\in\mathbb{R}\;\;\;\;\;\;\;\;\;\;\;\;\;\;\;\; (3)$$
Trong $(3)$ cho $x=-\dfrac{1}{2}$ ta được: $$\begin{aligned}&\;\;\;\;\;\;f(0)=2f\left(-\dfrac{y}{2}\right)+f(y)\\&\Leftrightarrow f(y)=2f\left(\dfrac{y}{2}\right) ,\;\forall y\in\mathbb{R}\end{aligned}$$ Kết hợp với $(3)$ ta được $$f(2xy+y)=f(2xy)+f(y),\,\forall x,y\in\mathbb{R}$$ Thay $2xy$ bởi $a$ và thay $y$ bởi $b$ từ đây ta được $$f(a+b)=f(a)+f(b),\;\forall a,b\in\mathbb{R}$$ Sử dụng cấu trúc nghiệm của phương trình hàm Cauchy ta tìm được $f(x)\equiv kx,\;\forall k,x\in\mathbb{R}$. Thử lại thấy thỏa. Kết luận: $$\boxed{f(x)\equiv kx,\;\forall k,x\in\mathbb{R}}$$

Monday, March 30, 2015

Hệ PT thi thử diễn đàn Mathlinks.vn

Bài toán: (Mathlinks.vn) Giải hệ phương trình: $$\begin{cases}(x+y)\left(\dfrac{1}{xy}+3\right)=\dfrac{6(x^2+y^2)+4}{\sqrt{2(x^2+y^2)}}\;\;\;\;\;\;\;\;\;\;\;\;\;\;\;\;\;\;\;(1)\\\\4-x^2-y^2=2\sqrt{2xy}+\sqrt{2-x^2-y^2}\;\;\;\;\;\;\;\;\;\;\;\;\;\;\;\;\;(2)\end{cases}$$ Lời giải: 

 ĐK: $\begin{cases}x^2+y^2\le 2\\x,y>0\end{cases}$. Bây giờ, ta sẽ chứng minh với điều kiện này thì: $$\begin{align}&\;\;\;\;\;\;\;(x+y)\left(\dfrac{1}{xy}+3\right)\ge\dfrac{6(x^2+y^2)+4}{\sqrt{2(x^2+y^2)}}\;\;\;\;\;\;\;\;\;\;\;\;\;\;\;\;\;\;(*) \\ & \Leftrightarrow \dfrac{x+y}{xy}+3(x+y)-\dfrac{6(x^2+y^2)+4}{\sqrt{2(x^2+y^2)}}\ge 0 \\ & \Leftrightarrow \dfrac{1}{x}+\dfrac{1}{y}-\dfrac{4}{x+y}+\dfrac{4}{x+y}-\dfrac{4}{\sqrt{2(x^2+y^2)}}+3(x+y)-3\sqrt{2(x^2+y^2)}\ge 0 \\ & \Leftrightarrow \dfrac{(x-y)^2}{xy(x+y)}+\dfrac{4(x-y)^2}{(x+y)\sqrt{2(x^2+y^2)}\left[ x+y+\sqrt{2(x^2+y^2)}\right] }-\dfrac{3(x-y)^2}{x+y+\sqrt{2(x^2+y^2)}}\ge 0 \\ & \Leftrightarrow \dfrac{1}{xy(x+y)}+\dfrac{4}{(x+y)\sqrt{2(x^2+y^2)}\left[ x+y+\sqrt{2(x^2+y^2)}\right] }-\dfrac{3}{x+y+\sqrt{2(x^2+y^2)}}\ge 0 \\ & \Leftrightarrow \dfrac{1}{xy(x+y)}-\dfrac{8}{(x+y)\sqrt{2(x^2+y^2)}\left[ x+y+\sqrt{2(x^2+y^2)}\right]}+\dfrac{12}{(x+y)\sqrt{2(x^2+y^2)}\left[ x+y+\sqrt{2(x^2+y^2)}\right]}-\dfrac{3}{x+y+\sqrt{2(x^2+y^2)}}\ge  0 \\ & \Leftrightarrow \dfrac{\sqrt{2(x^2+y^2)}\left[x+y+\sqrt{2(x^2+y^2)}\right] -8xy}{xy(x+y)\sqrt{2(x^2+y^2)}\left[ x+y+\sqrt{2(x^2+y^2)}\right]}+\dfrac{3.\left[ 4-(x+y)\sqrt{2(x^2+y^2)}\right]}{(x+y)\sqrt{2(x^2+y^2)}\left[ x+y+\sqrt{2(x^2+y^2)}\right]}\ge 0\end{align}$$ BDT này đúng vì theo Cauchy Schwarz và AM-GM ta có: $$\sqrt{2(x^2+y^2)}\left[x+y+\sqrt{2(x^2+y^2)}\right]\ge 2(x+y)^2\ge 8xy$$ Vì $x^2+y^2\le 2$ nên: $$4-(x+y)\sqrt{2(x^2+y^2)}\ge 4-2(x^2+y^2)\ge 0$$ BDT $(*)$ được chứng minh. Dấu "=" xảy ra khi và chỉ khi $x=y$. Thay vào phương trình $(2)$ ta được: $$\begin{aligned}&\;\;\;\;\;\; 4-2x^2=2x\sqrt{2}+\sqrt{2-2x^2}\\ & \Leftrightarrow 1-2x^2+2(1-x\sqrt{2})+1-\sqrt{2-2x^2}=0\\ & \Leftrightarrow (1-x\sqrt{2})\left[(1+x\sqrt{2}).\dfrac{\sqrt{2-2x^2}}{1+\sqrt{2-2x^2}}+2\right] =0\\ & \Leftrightarrow 1-x\sqrt{2}=0,\;\;\;\text{Vì }(1+x\sqrt{2})\dfrac{\sqrt{2-2x^2}}{1+\sqrt{2-2x^2}}+2 >0\\&\Leftrightarrow x=\dfrac{\sqrt{2}}{2} \end{aligned}$$ Cuối cùng, ta thu được nghiệm $$(x, y)=\left( \dfrac{\sqrt{2}}{2},\;\dfrac{\sqrt{2}}{2}\right)$$ 

Friday, March 27, 2015

Phương pháp S-S

Bài toán: (Toán học Tuổi trẻ) Kí hiệu $a, b, c, R, r$ lần lượt là độ dài ba cạnh, bán kính đường tròn ngoại tiếp và nội tiếp tam giác đó. Chứng minh rằng: $$\dfrac{r}{R}\le\dfrac{3(ab+bc+ca)}{2(a^2+b^2+c^2)}$$
Lời giải:
Ta có: $$\dfrac{r}{R}=\dfrac{\dfrac{S}{p}}{\dfrac{abc}{4S}}=\dfrac{4S}{pabc}=\dfrac{4p(p-a)(p-b)(p-c)}{pabc}=\dfrac{(b+c-a)(c+a-b)(a+b-c)}{2abc}$$ Do $a, b, c$ là độ dài ba cạnh một tam giác nên tồn tại $x,y, z>0$ sao cho $a=x+y, b=y+z, c=z+x$. Suy ra: $$\dfrac{r}{R}=\dfrac{8xyz}{2(x+y)(y+z)(z+x)}$$ $$\begin{aligned}\dfrac{ab+bc+ca}{2(a+b+c)^2}&=\dfrac{(x+y)(y+z)+(y+z)(z+x)+(z+x)(x+y)}{8(x+y+z)^2}\\&=\dfrac{x^2+y^2+z^2+3(xy+yz+zx)}{8(x+y+z)^2}\end{aligned}$$ Lúc này bất đẳng thức cần chứng minh trở thành: $$\begin{aligned}&\;\;\;\;\;\;\dfrac{8xyz}{2(x+y)(y+z)(z+x)}\le\dfrac{3[x^2+y^2+z^2+3(xy+yz+zx)]}{8(x+y+z)^2}\\&\Leftrightarrow \dfrac{2(x+y)(y+z)(z+x)}{8xyz}\ge \dfrac{8(x+y+z)^2}{3[x^2+y^2+z^2+3(xy+yz+zx)]}\\&\Leftrightarrow \dfrac{(x+y)(y+z)(z+x)-8xyz}{8xyz}\ge \dfrac{8(x+y+z)^2-6[x^2+y^2+z^2+3(xy+yz+zx)]}{3[x^2+y^2+z^2+3(xy+yz+zx)]}\\&\Leftrightarrow \dfrac{2z(x-y)^2+(x+y)(x-z)(y-z)}{8xyz}\ge\dfrac{(x-y)^2+(x-z)(y-z)}{3[x^2+y^2+z^2+3(xy+yz+zx)]}\\&\Leftrightarrow M(x-y)^2+N(x-z)(y-z)\ge 0\end{aligned}$$ Trong đó $$\begin{aligned} M&=\dfrac{2z}{8xyz}-\dfrac{1}{3[x^2+y^2+z^2+3(xy+yz+zx)]}\\&=\dfrac{z[x^2+y^2+z^2+3(xy+yz+zx)]-4xyz}{12xyz[x^2+y^2+z^2+3(xy+yz+zx)]}\\&=\dfrac{3z(x^2+y^2+z^2)+9z^2(y+x)+xyz}{12xyz[x^2+y^2+z^2+3(xy+yz+zx)]}>0\\\\N&=\dfrac{x+y}{8xyz}-\dfrac{1}{3[x^2+y^2+z^2+3(xy+yz+zx)]}\\&=\dfrac{3(x+y)[x^2+y^2+z^2+3(xy+yz+zx)]-8xyz}{24xyz[x^2+y^2+z^2+3(xy+yz+zx)]}\\&=\dfrac{3y[x^2+y^2+z^2+3(xy+yz+zx)]+3x(x^2+y^2+z^2)+9x^2(y+z)+xyz}{24xyz[x^2+y^2+z^2+3(xy+yz+zx)]}>0\end{aligned}$$ Đồng thời, không giảm tính tổng quát, ta có thể giả sử $z=\min\{x,y,z\}$. Như vậy, hiển nhiên ta có ngay: $$M(x-y)^2+N(x-z)(y-z)\ge 0$$ Bài toán được chứng minh.

Saturday, March 14, 2015

LỚP CÁC PHƯƠNG TRÌNH HÀM XOAY QUANH CÁC ĐẲNG THỨC LƯỢNG GIÁC

LỚP CÁC PHƯƠNG TRÌNH HÀM XOAY QUANH CÁC ĐẲNG THỨC LƯỢNG GIÁC


Ta chú ý tới các đẳng thức sau: $$\begin{aligned}&\sin (x+y)=\sin x \cos y+\sin y \cos x\;\;\;\;\;\;\;\;\;\;\;\;\;\;\;\;\;\;\;\;\;\;\;\;\;\;\;\;\;\;\;\;(1)\\& \sin (x-y)=\sin x\cos y-\sin y\cos x\;\;\;\;\;\;\;\;\;\;\;\;\;\;\;\;\;\;\;\;\;\;\;\;\;\;\;\;\;\;\;\;(2)\\&\cos (x+y)=\cos x\cos y-\sin x\sin y\;\;\;\;\;\;\;\;\;\;\;\;\;\;\;\;\;\;\;\;\;\;\;\;\;\;\;\;\;\;\;\;(3)\\&\cos (x-y)=\cos x\cos y+\sin x\sin y\;\;\;\;\;\;\;\;\;\;\;\;\;\;\;\;\;\;\;\;\;\;\;\;\;\;\;\;\;\;\;\;(4)\\&\sin (x+y)\sin (x-y)=\sin ^2x\cos ^2y-\sin ^2y\cos ^2 x\;\;\;\;\;\;\;\;\;\;\;(5)\end{aligned}$$

I. Phương trình hàm Cauchy:

   Bài toán 1:  Tìm tất cả hàm $f:\mathbb{R}\to \mathbb{R}$, liên tục trên $\mathbb{R}$ và thỏa: $$f(x+y)=f(x)+f(y),\forall x, y\in\mathbb{R}\;\;\;\;\;\;\;\;\;\;\;\;\;\;\;(1)$$


Lời giải:

Trong $(1)$ cho $x=y$, ta được: $$f(2x)=2f(x),\forall x\in\mathbb{R}\;\;\;\;\;\;\;\;\;\;\;\;\;\;\;\;\;\;(2)$$ Trong $(2)$ cho $x=0$ ta được $f(0)=0$. Bằng qui nạp, ta dễ dàng chứng minh được: $$f(nx)=n.f(x),\forall x\in\mathbb{R}, n\in\mathbb{N}\;\;\;\;\;\;\;\;\;\;\;(3)$$
Trong $(1)$, ta cho $y=-x$, đồng thời chú ý $f(0)=0$, ta được: $$f(-x)=-f(x),\forall x\in\mathbb{R}$$ Như vậy, với $n\in\mathbb{Z^-}$, ta cũng có: $$f(nx)=f(-n.(-x))=-n.f(-x)=nf(x),\forall x\in\mathbb{R}$$ Do vậy, ta có được: $$f(nx)=nf(x),\forall x\in\mathbb{R}, n\in\mathbb{Z}\;\;\;\;\;\;\;\;\;\;\;\;\;\;(4)$$ Với $n\in\mathbb{N}$ theo $(3)$ ta có: $$f(x)=f\left(n.\dfrac{1}{n}x\right)=n.f\left(\dfrac{1}{n}x\right)\Rightarrow f\left(\dfrac{1}{n}x\right)=\dfrac{1}{n}f(x)\;\;\;\;\;\;\;\;\;\;\;(5)$$ Từ $(4)$ và $(5)$ ta có: $$f\left(\dfrac{m}{n}x\right) =f\left(m.\dfrac{1}{n}x\right) =mf\left(\dfrac{1}{n}x\right) =\dfrac{m}{n}f(x), \forall x\in\mathbb{R}$$ Tức là: $$f(nx)=nf(x),\forall x\in\mathbb{R},n\in\mathbb{Q}\;\;\;\;\;\;\;\;\;\;\;\;\;\;(6)$$ Trong $(6)$ cho $x=1$ ta được $f(n)=f(1).n,\forall n\in\mathbb{Q}$. Với mỗi $x\in\mathbb{R}$, ta thấy luôn tồn tại dãy số $(n_k)$ sao cho $\lim_{x\to +\infty } n_k=x$. Do $f$ liên tục nên ta có: $$f(x)=f\left(\lim_{x\to +\infty }n_k\right) =\lim_{x\to +\infty }f(n_k)=\lim_{x\to +\infty } n_k f(1)=f(1)\lim_{x\to +\infty }n_k=f(1). x,\forall x\in\mathbb{R}$$ Vậy ta có: $$f(x)\equiv ax, \forall x\in\mathbb{R}, a=const $$

Tính chất: Giả sử hàm $f$ cộng tính trên $\mathbb{R}$, khi đó ta có: 
            1. $f$ là hàm lẻ trên $\mathbb{R}$
            2. $f(x)=f(1).x, \forall x\in\mathbb{Q}$
            3. $f$ đơn điệu trên $\mathbb{R}$ thì $f(x)=f(1).x, \forall x\in\mathbb{R}$


   Bài toán 2:  Tìm hàm $f:\mathbb{R}\to\mathbb{R}$ liên tục trên $\mathbb{R}$ và thỏa mãn: $$ f(x+y)=f(x).f(y),\forall x, y\in\mathbb{R}\;\;\;\;\;\;\;\;\;\;\;\;\;\;\;\;\;\;\;\;(1)$$

Lời giải:

Nếu $f$ hằng thì dễ thấy $f(x)\equiv 0, \forall x\in\mathbb{R}$ thỏa mãn bài toán. Ta xét bài toán khi $f(x)\not\equiv 0$ khác hằng. Khi đó, chọn $x_0$ sao cho $f(x_0)\neq 0$. Trong $(1)$, ta có: $$f(x_0)=f(x+x_0-x)=f(x).f(x_0-x)\neq 0, \forall x\in\mathbb{R}$$ Suy ra: $f(x)\neq 0,\forall x\in\mathbb{R}$ và $f(x)=f\left(\dfrac{x}{2}+\dfrac{x}{2}\right) =f^2\left( x\right)>0,\forall x\in\mathbb{R}$
Đặt $g(x)=\ln f(x)$. Khi đó từ $(1)$ ta có: $$g(x+y)=g(x)+g(y)$$ Sử dụng bài toán 1, ta có $g(x)\equiv ax, \forall x\in\mathbb{R}$. Từ đó suy ra: $f(x)= e^{ax},\forall x\in \mathbb{R}$. Thử lại thấy thỏa. Vậy có hai hàm thỏa bài toán: $$f(x)\equiv 0,\forall x\in\mathbb{R}\\f(x)\equiv a^x, \forall x\in\mathbb{R}, a>0$$


   Bài toán 3: Tìm hàm $f:\mathbb{R\setminus \{ 0\} }\to\mathbb{R}$, liên tục trên $\mathbb{R\setminus \{0\}}$ và thỏa: $$f(xy)=f(x)+f(y),\forall x, y\in\mathbb{R\setminus \{0\}}\;\;\;\;\;\;\;\;\;\;\;\;\;\;\;(1)$$

Lời giải:
Dễ thấy $f(x)\equiv 0,\forall x\in\mathbb{R}$ là một hàm thỏa $(1).
 Trường hợp 1: $f$ trên khoảng $(0; +\infty )$. Ta chọn $u, v$ sao cho $x=e^u, y=e^b$. Đặt $g(x)=f(e^x)$. Khi đó $(1)$ trở thành: $$g(u+v)=g(u)+g(v),\forall u,v \in\mathbb{R}$$ Sử dụng bài toán 1, ta có $g(x)=ax\Rightarrow f(x)=a.\ln x, \forall x>0, a\in\mathbb{R}$.
 Trường hợp 2: $f$ trên khoảng $(-\infty; 0)$. Trong $(1)$ cho $x=y$ ta được: $$f(x^2)=f^2(x),\forall x\in\mathbb{R^-} \\\Leftrightarrow f(x)=\dfrac{1}{2}f(x^2),\forall x\in\mathbb{R^-}$$ Thay vào $(1)$ và sử dụng kết quả từ trường hợp 1, ta thấy: $$f(x)=\dfrac{1}{2}f(x^2)=\dfrac{1}{2}b.\ln |x|, \forall x\in\mathbb{R^-}$$ Kết hợp cả hai trường hợp ta được $$f(x)=C.\ln |x|, \forall x\in\mathbb{R\setminus \{0\}}$$


   Bài toán 4:  Tìm hàm $f: \mathbb{R^+}\to\mathbb{R^+}$ thỏa: $$f(xy)=f(x).f(y),\forall x, y\in\mathbb{R^+}$$

Lời giải:

Đặt $f(x)= e^{g(x)}$ Ta có được: $$g(xy)=g(x)+g(y),\forall x, y\in\mathbb{ R^+}$$ Sử dụng bài toán 3, ta có $g(x)=C.\ln |x|\Rightarrow f(x)=a x, \forall x\in\mathbb{R^+}$, thử lại thấy thỏa. Ngoài ra dễ thấy $f(x)=0, f(x)=1.\forall x\in\mathbb{R^+}$ cũng là nghiệm của bài toán.


Nhận xét:
    Từ đẳng thức $(5)$, nếu ta xét hàm $f(x)=\sin x$ thì ta được: $$f(x+y).f(x-y)=f^2(x)-f^2(y),\forall x, y\in\mathbb{R}$$
    Cộng $(3)$ và $(4)$ lại với nhau, ta có: $$\cos (x+y)+\cos (x-y)=2\cos \cos y,\forall x, y\in\mathbb{R}$$ Từ đó, xét hàm $g(x)=\cos x$ ta được bài toán phương trình hàm D' Alembert: $$f(x+y)+f(x-y)=2f(x)f(y),\forall x, y\in\mathbb{R}$$
   Lấy $(4)-(3)$, ta được: $$g(x-y)-g(x+y)=2.\sqrt{(1-g^2(x))(1-g^2(y))},\forall x, y\in \mathbb{R}$$
 ......
 Tương tự, từ những đẳng thức $(1), (2), (3), (4), (5)$ và nhiều đẳng thức lượng giác khác ta có thể thấy những nét thú vị của nó qua từng bài toán.

II. Phương trình hàm D'Alembert:

  Bài toán: (Phương trình hàm D'Alembert) Tìm hàm $f:\mathbb{R}\to\mathbb{R}$ liên tục và thỏa: $$f(x+y)+f(x-y)=2f(x)f(y),\forall x, y\in\mathbb{R}\;\;\;\;\;\;\;\;\;\;\;\;\;\;\;\;\;\;\;\;(1)$$

Lời giải:
Trong $(1)$ cho $x=y=0$ ta tính được $f(0)=0\vee f(0)=1$. Nếu $f(0)=0$, trong $(1)$ cho $y=0$ ta tìm được $f(x)\equiv 0,\forall x\in\mathbb{R}$, thử lại thấy thỏa
Xét khi $f(0)=1$. Cho $x=y$ từ $(1)$ ta được: $$f(2x)+f(0)=2f^2(x)\Leftrightarrow f(2x)=2f^2(x)-1\ge -1,\forall x,\in\mathbb{R}\;\;\;\;\;\;\;\;\;\;\;\;\;\;\;(2)$$ Ta xét hai trường hợp:
    Trường hợp 1: $| f(x) |\le 1$ Do $f$ liên tục nên tồn tại $\alpha >0$ sao cho $f(x)>0, \forall x\in (-\alpha ,\alpha )$. Khi đó, chọn $n_0\in\mathbb{N}$ đủ lớn sao cho $$f\left(\dfrac{x}{2^{n_0}}\right) >0\Rightarrow f\left(\dfrac{x}{2^{n_0}}\right) < 1$$ Như vậy, tồn tại $x_0=\dfrac{x}{2^{n_0}}\neq 0$ để: $$0< f(x_0)<f(x),\forall x\in (-|x_0|;|x_0|), f(x_0)=\cos \beta ,\beta\in\left(0;\dfrac{\pi}{2}\right)$$ Bằng qui nạp và từ $(2)$ ta chứng minh được $$f(kx)=\cos k\beta,\forall k\in\mathbb{N^+}$$ đồng thời vì $f$ chẵn nên $$f(k x)=\cos \beta ,\forall k\in\mathbb{Z}$$ Do tính trù mật trong $\mathbb{R}$ nên $f(x)=\cos ax, a\neq 0$, thử lại thấy thỏa.
   Trường hợp 2: $| f(x) |> 1$. Ta thu được nghiệm bài toán là $f(x)= \cosh x$. Đây là hàm cosin hyprebol ta không khảo sát trong chương trình phổ thông (Có thể tham khảo tại đây )

Nhận xét: Từ bài toán trên ta có định lý về nghiệm của phương trình hàm D'Alembert:
Hàm $f:\mathbb{R}\to\mathbb{R}$ thỏa: $$f(x+y)+f(x-y)=2f(x)f(y)$$ thì: $$f(x)=0,\forall x\in\mathbb{R}\\f(x)=1,\forall x\in\mathbb{R}\\f(x)=\cos ax, a\neq 0,x\in\mathbb{R}\\f(x)=\cosh ax, a\neq 0, x\in\mathbb{R}$$

Mở rộng phương trình hàm D'Alembert:  Tìm hàm $f:\mathbb{R}\to\mathbb{R}$ thỏa: $$f(x-y+a)-f(x+y+a)=2f(x)f(y)$$ Đáp số: $f\equiv 0 \vee f(x)=\cos \left(\dfrac{\pi}{2a}(x-a)\right) ,\forall x\in\mathbb{R}$

                                                                                                                                            To be countinue....

 

Tuesday, March 10, 2015

Bài toán: (Mathlink.vn) Giải hệ phương trình: $$\begin{cases}\dfrac{1}{(\sqrt{x}+1)(\sqrt{y}+1)}=\dfrac{1+\sqrt{2-x^2-y^2}}{\sqrt{xy(2-x^2-y^2)}+4}\;\;\;\;\;\;\;\;\;\;\;\;\;(1)\\\\\sqrt{x}+\sqrt{y}+\sqrt{2-x^2-y^2}(\sqrt{xy}+1)=2\;\;\;\;\;\;\;\;\;\;\;\;\;\;\;(2)\end{cases}$$ 
Lời giải:
Điều kiện: $\begin{cases}x, y\ge 0\\2-x^2-y^2\ge 0\end{cases}\Rightarrow 0\le x,y\le \sqrt{2}$ Phương trình $(2)$ tương đương với: $$(\sqrt{x}+1)(\sqrt{y}+1)=3+\sqrt{xy}-\sqrt{2-x^2-y^2}(\sqrt{xy}+1)$$ Đặt $a=\sqrt{2-x^2-y^2}, b=\sqrt{xy},\;(0\le a,b;\; b\le 2,\;a\le \sqrt{2})$  Kết hợp với $(1)$ ta có: $$\begin{aligned}&\;\;\;\;\;\;\dfrac{1+a}{ab+4}=\dfrac{1}{3+b-a(b+1)}\\&\Leftrightarrow (a+1)[3+b-a(b+1)]=ab+4\\&\Leftrightarrow b=\dfrac{(a-1)^2}{1-a-a^2}\end{aligned}$$ Ngoài ra: 
 +) Phương trình $(1)$ cho ta: $$\begin{aligned}&\;\;\;\;\;\;\; (\sqrt{x}+1)(\sqrt{y}+1)=\dfrac{\sqrt{xy(2-x^2-y^2)}+4}{1+\sqrt{2-x^2-y^2}}\\&\Leftrightarrow \sqrt{xy}+\sqrt{x}+\sqrt{y}+1=\dfrac{\sqrt{xy(2-x^2-y^2)}+4}{1+\sqrt{2-x^2-y^2}}\;\;\;\;\;\;\;\;\;\;\;\;\;\;\;\;(3)\end{aligned}$$
 +) Phương trình $(2)$ cho ta: $$\sqrt{x}+\sqrt{y}=2-\sqrt{2-x^2-y^2}(\sqrt{xy}+1)\;\;\;\;\;\;\;\;\;\;\;\;\;(4)$$ Lấy $(3)-(4)$, ta được: $$\sqrt{xy}+1=\sqrt{2-x^2-y^2}(\sqrt{xy}+1)+\dfrac{\sqrt{xy(2-x^2-y^2)}+4}{1+\sqrt{2-x^2-y^2}}-2$$ hay: $$\begin{aligned}&\;\;\;\;\;\;b+1=a(b+1)+\dfrac{ab+4}{a+1}-2\\&\Leftrightarrow b=ab+a+\dfrac{ab+4}{a+1}-3\\&\Rightarrow \dfrac{(a-1)^2}{1-a-a^2}=\dfrac{a(a-1)^2}{1-a-a^2}+a+\dfrac{\dfrac{a(a-1)^2}{1-a-a^2}+4}{a+1}-3\\&\Leftrightarrow \dfrac{a^3-2a^2+a}{1-a-a^2}=0\Leftrightarrow a=0\vee a=1\end{aligned}$$

Trường hợp 1: $a=0\Rightarrow b=1$ Lúc này ta thu được; $$\begin{cases}x^2+y^2=2\\xy=1\end{cases}\Leftrightarrow x=y=1\;(x,y \ge 0)$$

Trường hợp 2: $a=1\Rightarrow b=0$ Lúc này ta được; $$\begin{cases}x^2+y^2=1\\xy=0\end{cases}\Leftrightarrow \begin{cases}x=0\\y=1\end{cases}\vee\begin{cases}x=1\\y=0\end{cases}$$ Thử lại thấy thỏa. Kết luận $$(x,y)=(1,1)\vee (0,1)\vee (1,0)$$

Wednesday, February 25, 2015

Bài toán: Tìm tất cả hàm số $f:\mathbb{R}\to\mathbb{R}$ thỏa mãn: $$f(x.f(y))+f(y.f(z))+f(z.f(x))=xy+yz+zx,\forall x, y, z\in\mathbb{R}\;\;\;\;\;\;\;\;(*)$$

Lời giải:
Trong $(*)$ cho $x=y=z=0$ ta được $f(0)=0$.
Tiếp tục cho $z=0$, từ $(*)$ ta được: $$f(xf(y))=xy,\forall x,y\in\mathbb{R}\;\;\;\;\;\;\;\;\;\;\;\;\;\;\;(1)$$ Trong $(1)$ cố định $x$, với mỗi $f(y_1)=f(y_2)$ ta có: $$f(xf(y_1))=f(xf(y_2))\Leftrightarrow xy_1=xy_2\Leftrightarrow y_1=y_2$$ Như vậy $f$ là đơn ánh. Ngoài ra trong $(1)$ cho $x=1$ ta được $f(f(x))=x,\forall x\in\mathbb{R}$, điều này chứng tỏ $f$ là toàn ánh.
Trong $(*)$ cho $x=1, y=f(1)$, ta được: $$f(f(f(1)))+f(f(1).z)+f(f(1)z)=f(1)+f(1)z+z,\forall z\in\mathbb{R}$$ Chú ý tới $(1)$ và $f$ là toàn ánh nên từ trên ta dễ dàng suy ra: $$f(f(1).z)=f(1).z,\forall z\in\mathbb{R}\;\;\;\;\;\;\;\;\;\;\;\;\;\;\;\;\;(2)$$ Ngoài ra, trong $(1)$ cho $y=1$ ta được: $$f(f(1).x)=x,\forall x\in\mathbb{R}\;\;\;\;\;\;\;\;\;\;\;\;\;\;\;(3)$$ Từ $(3)$ kết hợp với $(2)$ ta có được: $$x=f(1).x,\forall x\in\mathbb{R}\Rightarrow f(1)=1$$ Từ đây, kết hợp với $(3)$ ta được $f(x)\equiv x,\forall x\in\mathbb{R}$, tử lại thấy thỏa. Tóm lại, $$f(x)\equiv x,\forall x\in\mathbb{R}$$

Saturday, February 21, 2015

Bài toán: (Đề thi Olympic 30.4 lớp 11 năm 2014-2015)
        Tìm tất cả đa thức $P(x)$ hệ số thực thỏa: $$P(x).P(x^2)=P(x^3+3x),\forall x\in\mathbb{R}\;\;\;\;\;\;\;\;\;\;\; (*)$$

Lời giải:

Dễ thấy nếu $P(x)$ là đa thức hằng thì có hai đa thức thỏa mãn bài toán là $P(x)\equiv 0,\forall x\in\mathbb{R}$ và $P(x)\equiv 1,\forall x\in\mathbb{R}$. Ta xét trong trường hợp $P(x)$ khác hằng.
Trong $(*)$ cho $x=0$, ta được $P(0)=0$ hoặc $P(0)=1$.

    Trường hợp 1: $P(0)=0$. Đặt $P(x)=x^k.Q(x)$, trong đó $k\ge 1, Q(0)\neq 0$, thay vào $(*)$ ta được: $$x^{3k}Q(x).Q(x^2)=(x^3+3x)^k.Q(x^3+3x),\forall x\in\mathbb{R}\\\Leftrightarrow x^{2k}.Q(x).Q(x^2)=(x^2+3)^k.Q(x^3+3x),\forall x\in\mathbb{R^*}$$ Do $Q(0)\neq 0$ nên $Q(x)$ có hệ số tự do khác $0$. Gọi hệ số này là $d$. Ta nhận thấy:

       $\bullet$ Hệ số bậc nhỏ nhất của vế trái là: $d^2$
       $\bullet$ Hệ số bậc nhỏ nhất của vế phải là: $3^kd$
 Từ đó suy ra: $$d^2=3^k d\Leftrightarrow d=0, \text{ hay } Q(0)=0,\text{vô lý}$$ Do vậy, ta thu được $P(x)\equiv 0, \forall x\in\mathbb{R}$, thử lại thấy thỏa $(*)$.

    Trường hợp 2: $P(0)=1$. Đặt $P(x)=x^h.F(x)+1$ trong đó:  $h\ge 1, F(0)\neq 0$, thay vào $(*)$ ta được: $$x^{3h}F(x).F(x^2)+x^hF(x)+x^{2h}F(x^2)=(x^3+3x)^hF(x^3+3x),\forall x\in\mathbb{R}\\\Leftrightarrow x^{2h} F(x).F(x^2)+F(x)+x^hF(x^2)=(x^2+3)^h.F(x^3+3x),\forall x\neq 0$$ Do $F(0)\neq 0$ nên tồn tại $l\neq 0$ là hệ số tự do của $F(x)$. Tương tự như trường hợp 1, ta nhận thấy:

       $\bullet$ Hệ số bậc nhỏ nhất của vế trái là $l$
       $\bullet$ Hệ số bậc nhỏ nhất của vế phải là $3^h l$
  Từ đó suy ra: $$l=3^h.l\Leftrightarrow l=0,\text{ vô lý}$$
 Như vậy, ta thu được $P(x)\equiv 1, \forall x\in\mathbb{R}$, thử lại thấy thỏa $(*)$.
 Tóm lại, có hai đa thức thỏa mãn bài toán $$P(x)\equiv 0,\forall x\in\mathbb{R}\\\\P(x)\equiv 1,\forall x\in\mathbb{R}$$